2023 PREP Self-Assessment General Pediatrics Review

129 15 18MB

English Pages [909] Year 2023

Report DMCA / Copyright

DOWNLOAD FILE

Polecaj historie

2023 PREP Self-Assessment General Pediatrics Review

Table of contents :
Question 1
Question 2
Question 3
Question 4
Question 5
Question 6
Question 7
Question 8
Question 9
Question 10
Question 11
Question 12
Question 13
Question 14
Question 15
Question 16
Question 17
Question 18
Question 19
Question 20
Question 21
Question 22
Question 23
Question 24
Question 25
Question 26
Question 27
Question 28
Question 29
Question 30
Question 31
Question 32
Question 33
Question 34
Question 35
Question 36
Question 37
Question 38
Question 39
Question 40
Question 41
Question 42
Question 43
Question 44
Question 45
Question 46
Question 47
Question 48
Question 49
Question 50
Question 51
Question 52
Question 53
Question 54
Question 55
Question 56
Question 57
Question 58
Question 59
Question 60
Question 61
Question 62
Question 63
Question 64
Question 65
Question 66
Question 67
Question 68
Question 69
Question 70
Question 71
Question 72
Question 73
Question 74
Question 75
Question 76
Question 77
Question 78
Question 79
Question 80
Question 81
Question 82
Question 83
Question 84
Question 85
Question 86
Question 87
Question 88
Question 89
Question 90
Question 91
Question 92
Question 93
Question 94
Question 95
Question 96
Question 97
Question 98
Question 99
Question 100
Question 101
Question 102
Question 103
Question 104
Question 105
Question 106
Question 107
Question 108
Question 109
Question 110
Question 111
Question 112
Question 113
Question 114
Question 115
Question 116
Question 117
Question 118
Question 119
Question 120
Question 121
Question 122
Question 123
Question 124
Question 125
Question 126
Question 127
Question 128
Question 129
Question 130
Question 131
Question 132
Question 133
Question 134
Question 135
Question 136
Question 137
Question 139
Question 140
Question 141
Question 142
Question 143
Question 144
Question 145
Question 146
Question 147
Question 148
Question 149
Question 150
Question 151
Question 152
Question 153
Question 154
Question 155
Question 156
Question 157
Question 158
Question 159
Question 160
Question 161
Question 162
Question 163
Question 164
Question 165
Question 166
Question 167
Question 168
Question 169
Question 170
Question 171
Question 172
Question 173
Question 174
Question 175
Question 176
Question 177
Question 178
Question 179
Question 180
Question 181
Question 182
Question 183
Question 184
Question 185
Question 186
Question 187
Question 188
Question 189
Question 190
Question 191
Question 192
Question 193
Question 194
Question 195
Question 196
Question 197
Question 198
Question 199
Question 200
Question 201
Question 202
Question 203
Question 204
Question 205
Question 206
Question 207
Question 208
Question 209
Question 210
Question 211
Question 212
Question 213
Question 214
Question 215
Question 216
Question 217
Question 218
Question 219
Question 220
Question 221
Question 222
Question 223
Question 224
Question 225
Question 226
Question 227
Question 228
Question 229
Question 230
Question 231
Question 232
Question 233
Question 234
Question 235
Question 236
Question 237
Question 238
Question 239
Question 240
Question 241
Question 242
Question 243
Question 244
Question 245
Question 246
Question 247
Question 248
Question 249
Question 250
Question 251
Question 252
Question 253
Question 254
Question 255
Question 256
Question 257
Question 258
Question 259
Question 260
Question 261
Question 262
Question 263
Question 264
Question 265
Question 266
Question 267
Question 268
Question 269

Citation preview

PREP® Self-Assessment

American Academy of Pediatrics

PREPSA 2023

1

PREP® Self-Assessment

PREPSA 2023

Question 1 A 2-week-old boy is brought to the clinic for persistent vomiting. The neonate is exclusively fed a standard cow milk– based formula. He takes 4 oz every 2 hours and vomits after every feeding, despite burping. The emesis is not bloody, bilious, or projectile. He has 4 yellow, seedy, soft stools per day without blood. He is otherwise healthy. The neonate was born at term via an uncomplicated delivery and had an unremarkable newborn nursery course. His weight today is greater than his birth weight. His vital signs and physical examination findings are unremarkable. Of the following, the MOST likely cause of this neonate’s symptoms is A. gastroesophageal reflux disease B. milk protein allergy C. overfeeding D. pyloric stenosis

American Academy of Pediatrics

2

PREP® Self-Assessment

PREPSA 2023

Correct Answer: C The neonate in the vignette has gastroesophageal reflux (GER) caused by overfeeding. Gastroesophageal reflux, defined in the 2018 American Academy of Pediatrics Gastroesophageal Reflux Clinical Report (Eichenwald) as “the passage of gastric contents into the esophagus…can be distinguished from gastroesophageal reflux disease (GERD), which includes troublesome symptoms or complications associated with GER.” Gastroesophageal reflux is a relatively common, normal physiologic occurrence, affecting over two-thirds of healthy infants. Signs and symptoms of GER consist of spitting up, regurgitation, or even emesis. If there is associated pain, poor weight gain, or dysphagia, the infant would meet criteria for GERD; however, the neonate in the vignette does not have these symptoms. Overfeeding is a common cause of GER among infants. When the physiologic gastric volume capacity is exceeded, increased intragastric pressure results in GER. Although the volume consumed and timing of feedings varies among infants, full-term infants typically formula-feed 2 ounces every 3 to 4 hours (Item C1). The management of GER in infants includes reducing the feeding volume, increasing feeding frequency, avoiding seated and supine positions postfeedings, and assisting the infant in expelling gastric gas by patting the back (“burping”). Gastroesophageal reflux typically resolves by 12 months of age.

Cow milk protein allergy, or allergic proctocolitis, results from hypersensitivity to the proteins in cow milk–based formulas. Symptoms and signs commonly include fussiness; loose, mucousy stools; hematochezia; emesis; and/or GERD. Pyloric stenosis results from hypertrophy of the pylorus causing gastric obstruction. It typically presents between 3 and 5 weeks of age with forceful, often projectile, nonbloody, nonbilious emesis.

American Academy of Pediatrics

3

PREP® Self-Assessment

PREPSA 2023

PREP Pearls • Gastroesophageal reflux (GER) is a relatively common, normal physiologic occurrence, affecting over two-thirds of • healthy infants. Signs and symptoms of GER consist of spitting up, regurgitation, or even emesis. If there is associated pain, poor weight gain, or dysphagia, the infant would meet criteria for gastroesophageal reflux disease. • Overfeeding is a common cause of gastroesophageal reflux (GER) among infants. When the physiologic gastric volume capacity is exceeded, increased intragastric pressure results in GER. • The management of gastroesophageal reflux in infants includes reducing the feeding volume, increasing feeding frequency, avoiding seated and supine positions postfeedings, and assisting the infant in expelling gastric gas by patting the back (“burping”). Gastroesophageal reflux typically resolves by 12 months of age. ABP Content Specifications(s) • Differentiate gastroesophageal reflux from gastroesophageal reflux disease Suggested Readings • American Academy of Pediatrics Committee on Nutrition. Feeding the infant. In Kleinman RE, Greer FR, eds. Pediatric Nutrition. 7th ed. American Academy of Pediatrics; 2013:13-110. • DiMaggio DM, Cox A, Porto AF. Updates in infant nutrition. Pediatr Rev. 2017;38(10):449-462. doi:10.1542/pir.2016-0239 • Eichenwald EC; Committee on Fetus and Newborn; Cummings JJ, Aucott SW, Goldsmith JP, et al. Diagnosis and management of gastroesophageal reflux in preterm infants. Pediatrics. 2018;142(1):e20181061. doi:10.1542/peds.2018-1061 • Lightdale JR, Gremse DA; Section on Gastroenterology, Hepatoloy, and Nutrition. Gastroesophageal reflux: management guidance for the pediatrician. Pediatrics. 2013;131(5):e1684-e1695. doi:10.1542/peds.2013-0421. • Rosen R, Vandenplas Y, Singendonk M, et al. Pediatric gastroesophageal reflux clinical practice guidelines: joint recommendations of the North American Society for Pediatric Gastroenterology, Hepatology, and Nutrition and the European Society for Pediatric Gastroenterology, Hepatology, and Nutrition. J Pediatr Gastroenterol Nutr.2018;66(3):516-554. doi:10.1097/MPG.0000000000001889.

American Academy of Pediatrics

4

PREP® Self-Assessment

PREPSA 2023

Question 2 A 12-year-old, 40-kg boy in the pediatric intensive care unit with acute respiratory distress syndrome is currently intubated and mechanically ventilated, and has persistent hypoxemia. His last arterial blood gas analysis showed a pH of 7.22, pCO2 of 65 mm Hg, and a pO2 of 58 mm Hg. After adjusting the ventilator, his arterial blood gas analysis shows a pH of 7.25, pCO2 of 63 mm Hg, and pO2 of 80 mm Hg. Of the following, the MOST likely change made to the mechanical ventilator settings was a(n) A. decrease in positive end-expiratory pressure from 8 to 7 cm H2O B. decrease in tidal volume from 300 to 240 mL C. increase in inspiratory time from 0.7 to 0.9 seconds D. increase in respiratory rate from 22 to 24 breaths/minute

American Academy of Pediatrics

5

PREP® Self-Assessment

PREPSA 2023

Correct Answer: C The child in the vignette has acute respiratory distress syndrome (ARDS) and persistent hypoxemia. The change made to the mechanical ventilator settings resulted in an improvement in oxygenation (pO2) with negligible change in ventilation (pCO2). Oxygenation is a function of mean airway pressure. The main drivers of mean airway pressure are peak inspiratory pressure (PIP), positive end-expiratory pressure (PEEP), and inspiratory time fraction (i-time) (Item C2A). Thus, the most likely ventilator change that resulted in a higher pO2 was an increase in the inspiratory time fraction. Decreasing the PEEP would have resulted in the opposite effect and likely a lower pO2. A decrease in the tidal volume would not have increased the pO2 and would have caused an increase in pCO2. An increase in the respiratory rate would have little effect on oxygenation and would have decreased the pCO2.

Item C2A Determinants of oxygenation during pressure-limited, time-cycled ventilation. Shaded circles represent ventilator-controlled variables. Solid lines represent the simple mathematical relationships that determine mean airway pressure and oxygenation, and dashed lines represent relationships that cannot be quantified with a simple mathematical method. From Carlo WA, Greenough A, Chatburn RL. Advances in conventional mechanical ventilation. In: Boynton BR, Carlo WA, Jobe AH, eds. New Therapies for Neonatal Respiratory Failure. Boston, Mass: Cambridge University Press; 1994.

Children with acute lung disease require intubation and mechanical ventilation for 2 primary causes of respiratory failure: (1) failure of oxygenation and (2) failure of ventilation. The ventilator management of these 2 distinct pathologic states requires an understanding of the basic mechanics of ventilation and ventilator function. Failure of oxygenation or ventilation can occur as a result of new-onset lung disease, cardiac dysfunction, neurologic abnormalities, or multi-organ system failure. Examples of primary parenchymal lung disease include: • Pneumonia • ARDS • Inhalation injury • Chest trauma Drowning American Academy of Pediatrics

6

PREP® Self-Assessment • • • •

PREPSA 2023

Aspiration Chronic lung disease Pulmonary fibrosis Asthma

The goals of mechanical ventilation include optimizing gas exchange, reducing work of breathing, and limiting ongoing lung damage. Mechanical ventilation, if not prescribed correctly, can lead to progressive and additional lung injury as a result of excessive pressure or overdistension from high delivered tidal volume. Alveolar pressure is the main determinant of oxygenation. Because alveolar pressure cannot be directly measured, mean airway pressure measured by the mechanical ventilator can be used as a surrogate for mean alveolar pressure. Mean airway pressure can be visualized schematically as the area under the curve of a single breath cycle when ventilator pressure is plotted against time (Item C2B). Mean airway pressure can be increased by providing a higher PIP, higher PEEP, or longer inspiratory time. A higher mean airway pressure results in improved oxygenation.

Item C2B: Percentage change in pressure in relation to the time (in time constants) allowed for equilibration. As a longer time is allowed for equilibration, a higher percentage change in pressure will occur. The same rules govern the equilibration for step changes in volume Excessive mean airway pressure, however, can result in lung damage from high peak/plateau pressures (barotrauma) or high alveolar stretch and tidal volume (volutrauma). Children who are intubated, are mechanically ventilated, and have ongoing hypoxemia ultimately require a higher mean airway pressure in order to improve their oxygenation status. Although an increase in the fraction of inspired oxygen (FiO2) can temporarily improve oxygenation, high concentrations of oxygen are toxic to the alveoli due to the development of free radicals. Increasing the FiO2 can temporize hypoxia; however, ventilator adjustments that increase mean airway pressure should be made in order to optimize oxygen delivery while minimizing toxicity. Ventilation refers to the clearance of CO2 from the body and is a function of minute ventilation. Minute ventilation can be calculated as the mathematical product of respiratory rate and tidal volume (MV= RR × TV) (Item C2C). If CO2 production is relatively constant, manipulating the minute ventilation will have a predictable effect on the concentration of CO2 and rate of elimination. Under steady state CO2 production, changes in minute ventilation result in a linear and inversely proportional clearance of CO2. Thus, an increase in minute ventilation will result in American Academy of Pediatrics

7

PREP® Self-Assessment

PREPSA 2023

a proportional decrease in pCO2, and a decrease in minute ventilation will result in an increase in pCO2. Item C2C alveolar minute ventilation = (tidal volume - dead space) X frequency

PREP Pearls • Mean airway pressure is the driving force behind oxygenation in mechanically ventilated patients and is affected primarily by changes in peak inspiratory pressure, positive endexpiratory pressure, and inspiratory time fraction. • Carbon dioxide is eliminated in a linear fashion, inversely proportional to minute ventilation (minute ventilation = respiratory rate × tidal volume). • Mechanical ventilation can induce lung injury by both excessive pressure (barotrauma) and/or excessive volume (volutrauma). ABP Content Specifications(s) • Plan the appropriate ventilatory support for patients with various conditions Suggested Readings • Carlo WA, Ambalavanan N. Conventional mechanical ventilation: traditional and new strategies. Pediatr Rev. 1999;20(12):e117-e126. doi:10.1542/pir.20-12-e117. • Cheifetz IM. Invasive and noninvasive pediatric mechanical ventilation. Respir Care. 2003;48(4):442-458. http://rc.rcjournal.com/content/48/4/442.

American Academy of Pediatrics

8

PREP® Self-Assessment

PREPSA 2023

Question 3 A 13-month-old girl is seen for a health supervision visit. She was diagnosed with perinatally acquired HIV infection at 2 weeks of age and is currently receiving combination antiretroviral therapy. She has no history of opportunistic infections. She received her routine childhood immunizations at ages 2, 4, and 6 months including Haemophilus influenzae type b (Hib) conjugate vaccine and pneumococcal conjugate vaccine (PCV13). Her physical examination findings are normal. Laboratory data are notable for a CD4+ T lymphocyte count of 750 cells/µL (reference range for age 1-5 years, ≥1,000 cells/µL), CD4+ percentage of 25% (reference range for age 1-5 years, ≥30%), and an HIV viral load of 4,900 copies/mL. Of the following, in addition to diphtheria-tetanus-pertussis (DTaP), Hib, PCV13, and hepatitis A, the MOST appropriate vaccine(s) to administer today is A. measles-mumps-rubella B. measles-mumps-rubella and varicella C. measles-mumps-rubella-varicella D. varicella

American Academy of Pediatrics

9

PREP® Self-Assessment

PREPSA 2023

Correct Answer: B The girl in the vignette is living with HIV infection and has evidence of low-level immunosuppression based on the absence of opportunistic infections, a CD4+ T lymphocyte count greater than 500/µL, and CD4+ percentage greater than 22%. In this setting, both measlesmumps-rubella (MMR) and varicella vaccines are indicated. In addition, all inactivated vaccines, including diphtheria-tetanus-pertussis (DTaP), Haemophilus influenzae type b (Hib), pneumococcal conjugate vaccine 13 (PCV13), and hepatitis A vaccines, should be administered at this visit. Given the lack of safety data, children with HIV infection should not receive the quadrivalent measles-mumps-rubellavaricella (MMRV) vaccine. All infants with HIV infection should receive the rotavirus vaccine, irrespective of CD4+ Tlymphocyte count or percentage. Severe immunosuppression, as a general principle, is a contraindication to the administration of live-virus vaccines (eg, MMR, varicella, and MMRV). This principle applies to individuals with high-level immunosuppression categorized as follows: • Children ages 1 through 13 years living with HIV with a CD4+ T lymphocyte percentage of less than 15% • Adolescents age 14 years and older living with HIV with a CD4+ T lymphocyte count of less than 200/µL Children with cancer receiving chemotherapy • Children with combined B- and T-lymphocyte primary immunodeficiency disorders (eg, severe combined immunodeficiency disorder) • Receipt of daily high doses of corticosteroids (dose of ≥20 mg/day or ≥2 mg/kg per day for 2 weeks or longer) • Receipt of certain biologic agents (eg, tumor necrosis factor–antagonists, anti–Blymphocyte or anti–Tlymphocyte monoclonal antibodies) • Receipt of solid organ transplant in the past 2 months • Receipt of hematopoietic stem cell transplant within the past 2 months (and often for a longer duration) Children with immunosuppression should receive killed vaccines as routinely recommended (eg, Hib, pneumococcal, meningococcal, inactivated polio, diphtheria and tetanus toxoids and acellular pertussis vaccine, hepatitis A, hepatitis B, and inactivated influenza vaccine). PREP Pearls • Administration of live vaccines, such as measles-mumps-rubella (MMR) and varicella, is recommended for children and adolescents living with HIV infection who have low-level or no immunosuppression. • Children living with HIV with high-level immunosuppression (defined as a CD4+ T lymphocyte percentage of less than 15% in children aged 1 through 13 years, or a CD4+ T lymphocyte count of less than 200 cells/µL in adolescents aged 14 years and older) must not receive measles-mumps-rubella (MMR) or varicella vaccines. • Children with immunosuppression, including HIV infection, should receive killed vaccines. American Academy of Pediatrics 10

PREP® Self-Assessment

PREPSA 2023

ABP Content Specifications(s) • Plan an immunization regimen for a patient with an immune deficiency, including an immune deficiency as a result of chemotherapy Suggested Readings • American Academy of Pediatrics. Human immunodeficiency virus infection. In: Kimberlin DW, Barnett ED, Lynfield R, Sawyer MH, eds. Red Book: 2021-2024 Report of the Committee on Infectious Diseases. 32nd ed. American Academy of Pediatrics; 2021. Accessed September 1, 2022. Red Book Online. • American Academy of Pediatrics. Immunization and other considerations in immunocompromised children. In: Kimberlin DW, Barnett ED, Lynfield R, Sawyer MH, eds. Red Book: 2021-2024 Report of the Committee on Infectious Diseases. 32nd ed. American Academy of Pediatrics; 2021. Accessed September 1, 2022. Red Book Online. • Humiston SG, Atkinson WL, Rand C, Szilagyi PG. Immunizations. In: McInerny TK, Adam HM, Campbell DE, DeWitt TG, Foy JM, Kamat DM, eds. American Academy of Pediatrics Textbook of Pediatric Care. American Academy of Pediatrics; 2021:chap 20. Accessed September 1, 2022. Pediatric Care Online. • Rubin LG, Levin MJ, Ljungman P, et al. 2013 IDSA clinical practice guideline for vaccination of the immunocompromised host. Clin Infect Dis. 2014;58(3):309-318. doi:10.1093/cid/cit816.

American Academy of Pediatrics

11

PREP® Self-Assessment

PREPSA 2023

Question 4 A 3-year-old boy is evaluated for a 1-month history of intermittent fevers up to 40°C, night sweats, and 2.2-kg weight loss. He has no known sick contacts and has not traveled recently. The boy is afebrile. He is irritable but consolable and appears well nourished and well developed. There are multiple subcentimeter cervical lymph nodes palpable, more on the left side than the right, and a 4 × 3–cm, firm, immobile, nontender, left-sided, supraclavicular lymph node. His spleen is palpable 1 cm below the left costal margin. The remainder of his physical examination findings are unremarkable. Of the following, the BEST next step in this boy’s management is referral to a(n) A. emergency department B. gastroenterologist C. infectious disease specialist D. otolaryngologist

American Academy of Pediatrics

12

PREP® Self-Assessment

PREPSA 2023

Correct Answer: D The boy in the vignette has cervical and supraclavicular lymphadenopathy. Cervical lymphadenopathy is very common and has many possible etiologies. Supraclavicular lymphadenopathy, however, is highly concerning for malignancy (eg, lymphoma) and requires further investigation with a lymph node biopsy. Of the response choices, an otolaryngologist is best suited to perform a biopsy. Interventional radiologists and pediatric surgeons also perform lymph node biopsies, but these specialties are not among the response choices. Cervical lymphadenopathy is most often caused by a viral infection. This type of lymphadenopathy is usually transient, and the lymph nodes are small, mobile, and nontender. Occasionally, lymphadenitis may occur, leading to a larger lymph node that is usually tender. Lymphadenitis is treated with antibiotics. Further investigation is indicated if the lymphadenitis persists after treatment or worsens. The investigation of lymphadenopathy should include (a): • Complete blood count (CBC) with differential: The CBC may be normal in the setting of a malignancy. Leukocytosis with a left shift may indicate a bacterial infection. Lymphocytosis may indicate a viral infection. Some viral infections cause leukopenia and lymphopenia (eg, HIV). • Epstein-Barr virus and cytomegalovirus titers • Erythrocyte sedimentation rate and C-reactive protein—elevation supports inflammation or infection. • Liver transaminases—elevation supports a viral infection or infiltrative process. • Lactate dehydrogenase and uric acid levels—elevation indicates an increase in cell turnover, which is associated with malignancy. Imaging studies are helpful in the evaluation of lymphadenopathy. Ultrasonography can provide valuable information about the size and architecture of the lymph nodes. Chest radiography may identify a mediastinal mass. Computed tomography with contrast may identify an abscess. Although consultation with other subspecialists may be helpful, the immediate priority is to obtain a biopsy specimen for diagnostic evaluation by a pathologist. It is important to obtain sufficient tissue for diagnosis from the most pathologic part of the lymph node. For this reason, an excisional biopsy may be preferred over a fine-needle aspiration biopsy. This child’s clinical condition does not warrant urgent evaluation or management in an emergency department. PREP Pearls • Supraclavicular lymphadenopathy is usually pathologic and requires further investigation with a lymph node biopsy. • Persistent or worsening lymphadenopathy requires further investigation. ABP Content Specifications(s) • Plan the appropriate diagnostic evaluation of unexplained lymphadenopathy Recognize clinical findings associated with lymphoma American Academy of Pediatrics

13

PREP® Self-Assessment

PREPSA 2023

Suggested Readings • Friedmann AM. Evaluation and management of lymphadenopathy in children. Pediatr Rev. 2008;29(2):53-60. doi:10.1542/pir.29-2-53. • Sahai S. Lymphadenopathy. Pediatr Rev. 2013;34(5):216-227. doi:10.1542/pir.34-5-216. • Weinberg GA, Sefel GB, Hall CB. Lymphadenopathy. In: McInerny TK, Adam HM, Campbell DE, DeWitt TG, Foy JM, Kamat DM, eds. American Academy of Pediatrics Textbook of Pediatric Care. American Academy of Pediatrics; 2021:chap 175. Accessed September 1, 2022. Pediatric Care Online. • Weinstock MS, Patel NA, Smith LP. Pediatric cervical lymphadenopathy. Pediatr Rev. 2018;39(9):433-443. doi:10.1542/pir.2017-0249.

American Academy of Pediatrics

14

PREP® Self-Assessment

PREPSA 2023

Question 5 A 2-year-old girl with a noncontributory medical history is brought to the emergency department for intermittent abdominal pain over the past 12 hours. The pain, though progressively worsening, waxes and wanes. She had 3 episodes of nonbloody, nonbilious vomiting today. Over the past 6 hours, the girl has had a significantly decreased activity level and intermittent irritability. Her vital signs include a temperature of 37.9°C, heart rate of 130 beats/min, respiratory rate of 24 breaths/min, and blood pressure of 108/76 mm Hg. On physical examination, the girl appears tired and cries intermittently while drawing her legs in toward her chest. She has right lower quadrant tenderness without rebound or guarding. The remainder of her physical examination findings are unremarkable. Abdominal ultrasonography findings are shown in Item Q5.

Of the following, the BEST next step in this girl’s management is a(n) A. air enema procedure B. computed tomography of the abdomen C. emergency laparotomy D. upper gastrointestinal imaging with small bowel series

American Academy of Pediatrics

15

PREP® Self-Assessment

PREPSA 2023

Correct Answer: A The girl in the vignette has signs and symptoms concerning for intussusception. Abdominal ultrasonography demonstrates the characteristic “target sign” (Item C5), also known as a “bull’s eye” or “coiled spring,” which represents layering of intestine within the intestine. Because the girl is hemodynamically stable, performing an air enema is the best next management step. An air enema can be both diagnostic and therapeutic. If the air enema fails to reduce the intussusception, an emergency laparotomy may be necessary. Surgical intervention is indicated as the initial treatment if free air is visualized on imaging studies or there is evidence of peritonitis on physical examination.

Although computed tomography (CT) of the abdomen can be used to diagnose intussusception, it is not therapeutic and may require the child to be sedated. In addition, it exposes the child to unnecessary and significant amounts of radiation. Computed tomography is typically reserved for children with suspected intussusception in whom other imaging modalities fail to reveal a diagnosis or to identify a pathological lead point cause for intussusception. Upper gastrointestinal imaging with small bowel series under fluoroscopy is the study of choice for suspected malrotation with midgut volvulus, but it would not be appropriate in this scenario. Intussusception is the invagination of one part of the intestine into another part, most commonly occurring at the ileocecal junction (90% of cases). It most often occurs in children aged 3 months to 3 years; it is the most common cause of bowel obstruction in children younger than 2 years. Common symptoms of intussusception include sudden onset of severe, colicky abdominal pain that progressively worsens, irritability, intermittent inconsolable crying, emesis, lethargy, and altered mental status. In some instances, the lethargy may be profound and episodic, and it may be the only presenting symptom. Intussusception must always be included in the differential diagnosis for a child with altered mental status. Physical examination findings may include abdominal tenderness and a palpable, sausage-shaped mass in the right lower quadrant (~50%). Although up to 50% of children with intussusception have grossly bloody stools, the classic finding of “currant jelly” (blood mixed with mucus) stool is rare. Currant-jelly stool is a late and ominous finding, indicating bowel ischemia and necrosis. The classic triad of abdominal pain, currant-jelly stools, and a palpable abdominal mass is present in less than 15% of children at the time of presentation.

American Academy of Pediatrics

16

PREP® Self-Assessment

PREPSA 2023

PREP Pearls • The classic triad of abdominal pain, currant-jelly stools, and a palpable abdominal mass is present in less than 15% of children with intussusception. • An air enema can be both diagnostic and therapeutic for intussusception. • Extreme lethargy or altered mental status may be the only presenting symptom of intussusception. ABP Content Specifications(s) • Recognize the clinical features associated with intussusception, and manage appropriately Suggested Readings • Baker RD. Acute abdominal pain. Pediatr Rev. 2018;39(3):130-139. doi:10.1542/pir.2017-0089. • Chiu L, Dudgeon D, Stallion A. Gastrointestinal obstruction. In: McInerny TK, Adam HM, Campbell DE, DeWitt TG, Foy JM, Kamat DM, eds. American Academy of Pediatrics Textbook of Pediatric Care. 2nd ed. American Academy of Pediatrics; 2021:chap 257. Accessed September 1, 2022. Pediatric Care Online. • Gayle, T. Sudden altered mental status in an 11-month-old boy. Pediatr Rev. 2020;41(2):88-89. doi:10.1542/pir.2018-0108.

American Academy of Pediatrics

17

PREP® Self-Assessment

PREPSA 2023

Question 6 A 2-year-old girl was seen for a routine health supervision visit 2 weeks ago. At that time, her weight was inadvertently entered into the medical record in pounds but labeled as kilograms. The girl’s mother called the advice line last night because her daughter had a fever and was provided an acetaminophen dose based on the incorrect weight. The pediatrician noticed the error when reviewing the chart this morning and called the girl’s mother for follow-up. The girl received 1 dose of acetaminophen since the overnight phone call, is feeling better, and is not exhibiting negative consequences of the dosing error. The dose administered was well below a hepatotoxic level. Of the following, this event is BEST categorized as a/an A. adverse event B. medical error C. non-preventable adverse event D. sentinel event

American Academy of Pediatrics

18

PREP® Self-Assessment

PREPSA 2023

Correct Answer: B The girl in the vignette received a larger than appropriate dose of acetaminophen because of an error in the weight documentation. As this occurrence was a preventable error that did not result in patient harm, it is best classified as a medical error. The term adverse event implies that the patient was harmed as a result of an intervention. A sentinel event is an error that leads to a patient’s death or serious injury. Careful study is needed to better prevent medical errors. In order to thoroughly study medical error, it is important to have a universal terminology. A medical error is defined by the Institute of Medicine as “the failure to complete a planned action or the initiation of a wrong plan to achieve a specific goal.” An adverse event is an injury caused by the medical management as opposed to the patient’s underlying disease process. An adverse event causes harm to the patient whereas a medical error does not necessarily do so. Medication error is the most common type of medical error and can occur anywhere along the path of a medication reaching a patient (eg, prescribing, prescription-filling, administration). An adverse drug event is a medication error that results in patient injury; an adverse drug event is the most common type of adverse event. Adverse drug events can be further categorized as preventable, non-preventable, and potential. A preventable adverse drug event is one that could have been avoided, while a non-preventable adverse drug event could not have been avoided. For example, a patient is prescribed a medication to which they have a known allergy (preventable) versus a patient with no known allergy who develops an allergic reaction to a prescribed medication (nonpreventable). A potential adverse drug event, also known as a “near miss” event, is a medication error that places the patient at significant risk of injury but does not result in harm. A sentinel event is an unexpected event that results in death or serious injury, or the risk of death or serious injury. Not all sentinel events are the result of a medical error. A serious adverse drug event may meet criteria for a sentinel event. PREP Pearls • The Institute of Medicine defines a medical error as “the failure to complete a planned action or the initiation of a wrong plan to achieve a specific goal.” • An adverse event is an injury caused by the medical management as opposed to the patient’s underlying disease process. • A sentinel event is an unexpected event that results in death or serious injury, or the risk of death or serious injury. ABP Content Specifications(s) • Understand and apply the definition of a medical error • Understand and apply the definition of a near-miss event • Understand and apply the definition of a sentinel event • Understand and apply the definition of a preventable adverse event American Academy of Pediatrics

19

PREP® Self-Assessment •

PREPSA 2023

Understand and apply the definition of a non-preventable adverse event

Suggested Readings • Agency for Healthcare Research and Quality. Adverse events, near misses and errors. Patient Safety Network. September 7, 2019. Accessed September 1, 2022. https://psnet.ahrq.gov/primer/adverse-events-near-misses-and-errors. • Institute of Medicine (US) Committee on Quality of Health Care in America, Kohn LT, Corrigan JM, Donaldson MS, eds. To Err is Human: Building a Safer Health System. National Academies Press (US); 2000. doi:10.17226/9728. • Leonard, MS. Patient safety and quality improvement: medical errors and adverse events. Pediatr Rev. 2010;31(4):151158. doi:10.1542/pir.31-4-151. • Neuspiel DR. Medical errors, adverse events, and patient safety. In: McInerny TK, Adam HM, Campbell DE, DeWitt TG, Foy JM, Kamat DM, eds. American Academy of Pediatrics Textbook of Pediatric Care. American Academy of Pediatrics; 2021:chap 288. Accessed September 1, 2022. Pediatric Care Online.

American Academy of Pediatrics

20

PREP® Self-Assessment

PREPSA 2023

Question 7 A 3-month-old male infant born at term is brought to the emergency department via ambulance for a seizure. Initial evaluation shows an ionized calcium level of 3.3 mg/dL (0.8 mmol/L) (reference range, 4.5-5.3 mg/dL [1.1-1.3 mmol/L]). He is treated with intravenous calcium, which aborts the seizure. The infant’s parents report that he has been jittery and irritable for the past few days. He exclusively breastfed for the first 2 months after birth. Due to fussiness, his parents recently started making formula using a recipe found on the internet containing hemp seed hearts, coconut water, dates, and sea moss. The infant was recently diagnosed with laryngomalacia after an evaluation for noisy breathing. His physical examination findings are normal for age. His parents are of normal stature. Laboratory evaluation drawn prior to treatment reveals the following: Laboratory Test Result Total calcium 4.5 mg/dL (1.1 mmol/L) (reference range, 9-11 mg/dL [2.2-2.8 mmol/L]) Phosphorous 1.2 mg/dL (reference range, 2.7-4.5 mg/dL) Magnesium 2.1 mg/dL (0.9 mmol/L) (1.6-2.6 mg/dL [0.71.1 mmol/L]) Parathyroid hormone 534 pg/mL (reference range, 10-65 pg/mL) Alkaline phosphatase 1,021 U/L (reference range, 146-477 U/L) 25-hydroxyvitamin D Pending 1,25-dihydroxyvitamin D Pending Of the following, the BEST next step in this infant’s management is oral administration of A. calcium, cholecalciferol, and calcitriol B. calcium and phosphorus C. magnesium D. phosphorus and calcitriol

American Academy of Pediatrics

21

PREP® Self-Assessment

PREPSA 2023

Correct Answer: A The infant in the vignette has hypocalcemia due to severe vitamin D deficiency. His noisy breathing and seizure are manifestations of hypocalcemia. After treating his symptomatic hypocalcemia with intravenous calcium, the infant should be treated with oral calcium and cholecalciferol (dietary vitamin D3). Calcitriol (1,25-dihydroxyvitamin D, the active form of vitamin D) should be added given the severity of his hypocalcemia. Calcitriol acts immediately to absorb calcium from the intestine while vitamin D stores are replenished by cholecalciferol. The infant should also be switched to a cow milk–based formula, given its higher vitamin D content, once he can safely drink from a bottle. Breast milk does not provide adequate vitamin D; infants who are exclusively or predominantly breastfed must receive supplementation with this vitamin. Although commercial formulas are fortified with vitamin D, homemade formulas, including the one described in the vignette, do not contain additional vitamin D. The American Academy of Pediatrics recommends that breastfed infants receive at least 400 IU of supplemental vitamin D daily. Vitamin D is a steroid hormone that is integral in the regulation of calcium and phosphorus homeostasis. Item C7A outlines vitamin D metabolism and action. Laboratory findings depend on the severity and duration of the vitamin D deficiency (Item C7B). In mild vitamin D deficiency, relative hypocalcemia will result in a rise in parathyroid hormone (PTH) to maintain serum calcium in the normal range. Calcium is reabsorbed from the kidney, absorbed from the gut (via conversion of remaining 25-hydroxyvitamin D stores to 1,25-dihydroxyvitamin D), and released from the bones. As vitamin D deficiency becomes more severe and PTH rises higher, calcium is depleted and levels begin to fall. Increased PTH levels cause phosphate wasting in the kidney, resulting in low serum phosphorus levels. Parathyroid hormone has an indirect effect on osteoclasts, resulting in increased bone resorption. Alkaline phosphatase levels rise as bone turnover is increased. When 25-hydroxyvitamin D stores are extremely low, there is inadequate vitamin D available for conversion to 1,25-dihydroxyvitamin D, which results in low levels of this hormone. The infant in the vignette is expected to have low 25-hydroxyvitamin D and 1,25dihydroxyvitamin D levels.

Item C7B

American Academy of Pediatrics

22

PREP® Self-Assessment

PREPSA 2023

Item C7A

Manifestations of severe vitamin D deficiency include rickets (widening of the wrists, rachitic rosary, craniotabes) and symptoms of hypocalcemia. Infants may display irritability, constipation, noisy breathing due to laryngospasm, and seizures. Older children may experience paresthesias. Treatment with phosphorus and calcitriol is indicated for hypophosphatemia due to defects in the fibroblast growth factor 23 (FGF23) pathway, usually due to mutations in the PHEX gene. Fibroblast growth factor 23 is the main hormone responsible for phosphorus wasting in the kidney. Burosumab, a monoclonal antibody that inhibits FGF23, is approved by the US Food and Drug Administration for the treatment of children aged 6 months and older with X-linked hypophosphatemia. Although the infant in the vignette is hypophosphatemic, this presentation is due to secondary hyperparathyroidism and the phosphate wasting effect that the elevated PTH level has on the kidney. Additionally, both parents are of normal stature, which is evidence against familial hypophosphatemia, an X-linked dominant disorder that negatively impacts growth. Magnesium is required for a normal PTH response to low calcium levels. In children with hypomagnesemia, hypocalcemia may develop due to inadequate PTH action. In this case, the PTH level would be inappropriately low. This situation commonly occurs in infants of diabetic mothers. American Academy of Pediatrics

23

PREP® Self-Assessment

PREPSA 2023

Calcium and phosphorus is the treatment for metabolic bone disease of prematurity, which occurs due to a deficiency of these minerals. The infant in the vignette was born at term, so this would be an unlikely diagnosis. In addition, severe hypocalcemia is an unlikely finding in metabolic bone disease of prematurity. PREP Pearls • The American Academy of Pediatrics recommends supplementation of 400 IU daily of vitamin D for exclusively breastfed infants. • Manifestations of severe vitamin D deficiency may include rickets (widening of the wrists, rachitic rosary, craniotabes) and hypocalcemia with secondary hyperparathyroidism. • Homemade infant formulas may contain or lack ingredients that may affect mineral homeostasis. ABP Content Specifications(s) • Recognize the effects of vitamin D deficiency in patients of various ages, including those who are breast-fed Suggested Readings • Dawodu A, Wagner CL. Vitamin D inadequacy. In: McInerny TK, Adam HM, Campbell DE, DeWitt TG, Foy JM, Kamat DM, eds. American Academy of Pediatrics Textbook of Pediatric Care. American Academy of Pediatrics; 2021:chap 346. Accessed September 1, 2022. Pediatric Care Online. • Holick MF, Binkley NC, Bischoff-Ferrari HA, et al; Endocrine Society. Evaluation, treatment, and prevention of vitamin D deficiency: an Endocrine Society clinical practice guideline. J Clin Endocrinol Metab. 2011;96(7):1911-30. doi:10.1210/jc.2011-0385. • Misra M, Pacaud D, Petryk A, Collett-Solberg PF, Kappy M. Vitamin D deficiency in children and its management: review of current knowledge and recommendations. Pediatrics. 2008; 122(2):398-417. doi:10.1542/peds.2007-1894. • Munns CF, Shaw N, Kiely M, et al. Global consensus recommendations on prevention and management of nutritional rickets. J Clin Endocrinol Metab. 2016;101(2):394-415. doi:10.1210/jc.2015-2175. • Vieira MA, Kube PK, van Helmond JL, et al. Recipe for disaster: homemade formula leading to severe complications in 2 infants. Pediatrics. 2021;148(3):e2021050947. doi:10.1542/peds.2021-050947. • Wagner CL, Greer FR; American Academy of Pediatrics Section on Breastfeeding; American Academy of Pediatrics Committee on Nutrition. Prevention of rickets and vitamin D deficiency in infants, children, and adolescents. Pediatrics. 2008;122(5):11421152. doi:10.1542/peds.2008-1862.

American Academy of Pediatrics

24

PREP® Self-Assessment

PREPSA 2023

Question 8 A 12-day-old male neonate in the neonatal intensive care unit develops apnea, bradycardia, and oxygen desaturation requiring an increase in continuous positive airway pressure ventilation (CPAP) from 6 to 8 cm H2O, and fraction of inspired oxygen (FiO2) from 0.23 to 0.40. His mean upper limb cuff blood pressure has ranged from 18 to 25 mm Hg for the past hour. The neonate was born at 26 weeks’ gestation with a birthweight of 940 g. Delivery was by cesarean section due to maternal preeclampsia with rupture of membranes at delivery. The neonate has been tolerating nasogastric feedings of 20 calorie/oz breast milk at 80 mL/kg/day and receiving parenteral nutrition through a central intravenous line. Laboratory data are shown: Laboratory Test Arterial blood gas base deficit White blood cell count Neutrophils Bands Lymphocytes Monocytes Eosinophils Hemoglobin Hematocrit Platelet count

Result 10 22,000/µL (22.0 × 10 9/L) 40% 25% 20% 11% 4% 10.8 g/dL (108 g/L) 30.5% 87 × 103/µL (87 × 10 9/L)

Blood cultures are pending. Of the following, the BEST treatment for this neonate is intravenous A. ampicillin and cefotaxime B. ampicillin and gentamicin C. meropenem and oxacillin D. vancomycin and gentamicin

American Academy of Pediatrics

25

PREP® Self-Assessment

PREPSA 2023

Correct Answer: D The preterm neonate in the vignette has signs and symptoms of late-onset sepsis with clinical deterioration and an elevated white blood cell count with a left shift. The most likely causative organisms are coagulase-negative staphylococci and gram-negative bacteria. Of the response choices, the combination of vancomycin (effective against coagulase-negative staphylococci) and gentamicin (effective against gram-negative bacteria such as Escherichia coli and Klebsiella pneumoniae) is the best treatment pending blood culture results. Ampicillin, gentamicin, cefotaxime, and meropenem are all effective against gram-negative bacteria, but they are not effective against coagulase-negative staphylococci. Oxacillin is also not effective against coagulase-negative staphylococci. Ampicillin and gentamicin is the empiric antibiotic combination of choice in early-onset sepsis and is effective against the common bacterial causes of early onset sepsis (eg, group B Streptococcus, E coli, and Listeria monocytogenes). This neonate’s risk factors for sepsis include extreme prematurity, presence of an endotracheal tube and central line, and parenteral nutrition. Extremely preterm neonates in the neonatal intensive care unit are at increased risk for infection due to immature defense barriers (eg, skin and mucous membranes), an immature immune system, invasive interventions (eg, mechanical ventilation, intravascular catheters, and parenteral nutrition), and comorbidities (eg, patent ductus arteriosus, necrotizing enterocolitis, and chronic lung disease). The more premature the neonate, the higher the risk of infection. Sepsis should always be considered as a cause of clinical deterioration in a preterm neonate. Neonatal sepsis is classified as early-or late-onset. Early-onset sepsis is defined as sepsis in the first week after birth, and late-onset sepsis occurs 7 or more days after birth. Early-onset sepsis is most often caused by maternal vertical transmission of microorganisms; late-onset sepsis is most often postnatally acquired. PREP Pearls • Risk factors for late-onset sepsis in preterm neonates include immature defense barriers, an immature immune system, invasive interventions, and comorbidities of prematurity. • The choice of empiric antibiotics for late-onset sepsis in a preterm neonate should be based on the most likely causative organisms, which include coagulase-negative staphylococci and gram-negative bacteria. ABP Content Specifications(s) • Plan appropriate antimicrobial therapy for suspected sepsis in the immediate newborn period Suggested Readings • Bentlin MR, Rugolo, LMSS. Late-onset sepsis: epidemiology, evaluation and outcome. NeoReviews. 2010;11(8):e426e435. doi:10.1542/neo.11-8-e426. American Academy of Pediatrics

26

PREP® Self-Assessment • • •



PREPSA 2023

Camacho-Gonzalez A, Spearman PW, Stoll BJ. Neonatal infectious diseases: evaluation of neonatal sepsis. Pediatr Clin North Am. 2013;60(2):367-389. doi:10.1016/j.pcl.2012.12.003. Chu A, Hageman JR, Schreiber M, Alexander K. Antimicrobial therapy and late-onset sepsis. NeoReviews. 2012;13(2):e94-e102. doi:10.1542/neo.13-2-e94. Kojaoghlanian T. The newborn at risk of infection. In: McInerny TK, Adam HM, Campbell DE, DeWitt TG, Foy JM, Kamat DM, eds. American Academy of Pediatrics Textbook of Pediatric Care. American Academy of Pediatrics; 2021:chap 102. Accessed September 1, 2022. Pediatric Care Online. Puopolo KM, Benitz WE, Zaoutis TE; Committee on Fetus and Newborn, Committee on Infectious Diseases. Management of neonates born at less than or equal to 34 6/7 weeks' gestation with suspected or proven early-onset bacterial sepsis. Pediatrics. 2018;142(6):e20182896. doi:10.1542/peds.2018-2896.

American Academy of Pediatrics

27

PREP® Self-Assessment

PREPSA 2023

Question 9 The mother of a 10-year-old girl with cystic fibrosis (CF), jointly managed by her pediatrician and a multidisciplinary CF center at a nearby children’s hospital, calls for advice about a positive airway culture. The girl was diagnosed with CF after a positive newborn screening result. She has been very healthy and has required no aggressive intervention for pulmonary infection. Her routine airway cultures, obtained in the CF center by deep throat/gag swab, have always grown normal flora or methicillin-sensitive Staphylococcus aureus. Her most recent culture was positive for Pseudomonas aeruginosa. The girl has no symptoms of illness. She is eating well and has had no weight loss. She is attending school regularly. Of the following, the MOST appropriate management strategy in response to this girl’s laboratory finding is A. hospitalization for intravenous antibiotic administration B. inhaled antibiotic treatment for eradication of the identified organism C. no intervention while she is asymptomatic D. repeat airway culture before implementing treatment

American Academy of Pediatrics

28

PREP® Self-Assessment

PREPSA 2023

Correct Answer: B The most appropriate management strategy for the girl in the vignette, who is experiencing her first acquisition of Pseudomonas aeruginosa infection, is an inhaled anti-pseudomonal antibiotic. Multiple studies demonstrate that proactive treatment to eradicate Pseudomonas from the airway in children with cystic fibrosis (CF) helps preserve lung function. A single 28-day treatment course of inhaled tobramycin given twice daily, with follow-up culture to document resolution and re-treatment if the culture is still positive, is the standard of care in the United States. The addition of oral antibiotics for acute eradication has not been shown to be more effective than an inhaled antibiotic alone. However, concurrent oral azithromycin has been demonstrated to delay symptomatic pulmonary exacerbation of cystic fibrosis in children with first acquisition of Pseudomonas. There are no data to support hospitalization for treatment with intravenous antibiotics as more effective than treatment with an inhaled antibiotic alone for eradication of Pseudomonas in a child with asymptomatic first acquisition. Because longitudinal epidemiologic studies demonstrate that acquisition of P aeruginosa as a chronic colonizing agent is associated with more rapid decline in lung function, quarterly airway cultures are the standard of care for individuals with CF. Even though the girl in the vignette is asymptomatic, she should be treated for eradication promptly on identification of P aeruginosa in her respiratory culture. There is no need to repeat the culture before treatment, but it should be repeated after the treatment to document eradication or determine the need for retreatment. There are no clear criteria defining the number of positive cultures required to diagnose chronic P aeruginosa colonization, for which treatment includes long-term, alternate-month, inhaled tobramycin. Nor are there clear guidelines for when to stop long-term treatment after some period of negative cultures. With the advent of highly effective modulator therapies, the incidence of P aeruginosa infection in individuals with CF has decreased, and many of those who had been chronically colonized become decolonized. How, if, and when to stop chronic inhaled antibiotic treatment in this population is unclear and requires ongoing investigation. Chronic lung disease with recurrent bronchitis leading to bronchiectasis is the most common respiratory complication of CF. The changing pattern of respiratory microorganisms cultured from individuals with CF over the years is shown in Item C9. Chronic colonization with methicillin-resistant Staphylococcus aureus (MRSA) has also been associated with a decline in lung function in individuals with CF. Several studies have investigated the eradication of MRSA airway infection, but there is no current widely accepted practice guideline recommending a standardized approach.

American Academy of Pediatrics

29

PREP® Self-Assessment

PREPSA 2023

Exocrine pancreatic insufficiency leading to malabsorption and malnutrition is the most common gastrointestinal complication of CF, present in about 85% of affected individuals. As children with CF age into adolescence and adulthood, the incidence of endocrine pancreatic dysfunction increases and manifests as CF-related diabetes (CFRD). Although insulin is usually required for management, CFRD differs from both type 1 and type 2 diabetes. Calorie restriction is not appropriate in the management of CFRD, and weight loss does not improve outcomes. There is no genetic predisposition to CFRD. However, some individuals with CF carry genetic modifiers that have overlap with those of type 2 diabetes. Cystic fibrosis manifestations and complications can affect the respiratory, gastrointestinal, and reproductive systems. PREP Pearls • Chronic colonization with Pseudomonas aeruginosa and/or methicillin-resistant Staphylococcus aureus is associated with a decline in lung function in children and adolescents with cystic fibrosis. • The most appropriate treatment of first acquisition of Pseudomonas in an individual with cystic fibrosis is 28 days of inhaled antipseudomonal antibiotic, usually tobramycin. • Cystic fibrosis-related diabetes is a later-onset complication of progressive pancreatic fibrosis. ABP Content Specifications(s) • Recognize the pulmonary and extrapulmonary complications of cystic fibrosis in children of various ages • Recognize the pathogens commonly associated with the pulmonary complications of cystic fibrosis American Academy of Pediatrics 30

PREP® Self-Assessment •

PREPSA 2023

Identify the age-related gastrointestinal and hepatobiliary features in a patient with cystic fibrosis

Suggested Readings • Dasenbrook EC, Checkley W, Mero CAl, et al. Association between respiratory tract methicillin-resistant Staphylococcus aureus and survival in cystic fibrosis. JAMA. 2010;303(23):2386-2392. doi:10.1001/jama.2010.791. • Jackson L, Waters V. Factors influencing the acquisition and eradication of early Pseudomonas aeruginosa infection in cystic fibrosis. J Cyst Fibros. 2021;20(1):8-16. doi:10.1016/j.jcf.2020.10.008. • Mogayzel PJ Jr, Naureckas ET, Robinson KA, et al; Cystic Fibrosis Foundation Pulmonary Clinical Practice Guidelines Committee. Cystic Fibrosis Foundation pulmonary guideline: pharmacologic approaches to prevention and eradication of initial Pseudomonas aeruginosa infection. Ann Am Thorac Soc. 2014;11(10):1640-1650. doi:10.1513/AnnalsATS.201404166OC. • Paranjape SM, Mogayzel PJ Jr. Cystic fibrosis. Pediatr Rev. 2014;35(5):194-205. doi:10.1542/pir.35-5-194. • Willey-Courand DB, Marshall BC. Cystic fibrosis. In: McInerny TK, Adam HM, Campbell DE, DeWitt TG, Foy JM, Kamat DM, eds. American Academy of Pediatrics Textbook of Pediatric Care. American Academy of Pediatrics; 2021:chap 239. Accessed September 1, 2022. Pediatric Care Online.

American Academy of Pediatrics

31

PREP® Self-Assessment

PREPSA 2023

Question 10 A 5-year-old girl is seen for a health supervision visit. She attends kindergarten and is doing very well. Her medical history is significant for recurrent joint subluxations of the hips, shoulders, knees, and elbows bilaterally, easy bruising, and poor wound healing often requiring placement of sutures to achieve closure. The family history is significant for similar findings in her father and grandfather. The girl’s father had a recurrent incisional hernia at an appendectomy site. There is no family history of arterial aneurysm or rupture. On physical examination, there are bruises on her lower extremities, widened atrophic scars with evidence of previous sutures (Item Q10), and skin and joint hypermobility. The remainder of her physical examination findings are normal.

Of the following, the girl’s MOST likely diagnosis is A. Ehlers-Danlos syndrome B. homocystinuria C. Loeys-Dietz syndrome D. Marfan syndrome

American Academy of Pediatrics

32

PREP® Self-Assessment

PREPSA 2023

Correct Answer: A The girl in the vignette has classical Ehlers-Danlos syndrome (cEDS). Ehlers-Danlos syndrome is a group of inherited connective tissue disorders. Cardinal features of EDS include joint hypermobility, skin hyperextensibility, and tissue fragility. The 2017 international classification describes 13 subtypes of EDS. The most common forms are classical, vascular, and hypermobile. There is wide phenotypic variability and genetic heterogeneity among the different EDS subtypes. The underlying genetic etiology is known for all subtypes of EDS except hypermobile EDS. Classical Ehlers-Danlos syndrome is inherited in an autosomal dominant manner. The clinical diagnosis of cEDS is made based on the presence of major and minor criteria. Major criteria include: • Skin hyperextensibility: assessed by pinching and lifting skin on the volar surface at the middle of the nondominant forearm. Skin is called hyperextensible if it can be stretched to 1.5 cm at the forearm and dorsum of hands, and to 3 cm for neck, elbows, and knees. • Atrophic scarring • Generalized joint hypermobility: assessed using the Beighton criteria for joint hypermobility. Joint hypermobility is defined as a score of 5 or above with a total score of 9 (Item C10A).

Minor criteria include: • Easy bruising • Epicanthal folds • Family history (first-degree relative meeting clinical criteria) • Hernia • Joint hypermobility-related complications (eg, sprains, subluxation, pain, flexible flatfoot) • Molluscoid pseudotumors (fleshy lesions associated with scars over pressure points such as the elbows and knees) • Skin fragility • Soft, doughy skin American Academy of Pediatrics

33

PREP® Self-Assessment •

PREPSA 2023

Subcutaneous spheroids (mobile spherical hard bodies that are palpable on the forearms and shins)

The minimal clinical diagnostic criteria suggestive of the diagnosis of cEDS include: • Presence of skin hyperextensibility and atrophic scarring; plus • Generalized joint hypermobility; or • At least 3 minor criteria The diagnosis of cEDS is confirmed by the presence of minimal clinical criteria and the identification of a pathogenic variant in one of the genes known to be associated with cEDS (COL5A1, COL5A2, COL1A1). Disorders of connective tissue can have considerable clinical overlap. While the absence of a pathogenic variant in one of the above genes does not exclude the diagnosis of cEDS, it suggests consideration of another underlying etiology. Evaluation and management for the established diagnosis of cEDS are listed in Item C10B.

Homocystinuria is characterized by the following features: • Central nervous system: developmental delay, intellectual disability • Eyes: ectopia lentis, severe myopia • Vascular system: thromboembolic episodes • Skeletal: tall stature, long limbs, scoliosis, pectus excavatum Homocystinura is inherited in an autosomal recessive manner. Atrophic scars, easy bruising, and joint hypermobility are not features of homocystinuria. Developmental delay is a universal finding, often the first presenting feature; this is not seen in the girl in the vignette. Clinical features of Loeys-Dietz syndrome are described below: • Craniofacial: hypertelorism, bifid uvula/cleft palate, strabismus, craniosynostosis • Cutaneous: velvety, translucent skin, easy bruising, dystrophic scars • Skeletal: pectus excavatum/carinatum, scoliosis, joint hypermobility, arachnodactyly, cervical spine malformation, clubfeet American Academy of Pediatrics

34

PREP® Self-Assessment •

PREPSA 2023

Vascular: cerebral, thoracic, and abdominal arterial aneurysms and/or dissections

Affected individuals are at increased risk of complications related to arterial aneurysms. While there is some clinical overlap with EDS, distinguishing features of Loeys-Dietz syndrome include craniofacial anomalies, pectus deformity, arachnodactyly, spine anomalies, and arterial aneurysms. Clinical features of Marfan syndrome are described below: • Cardiovascular: aortic dilatation, mitral valve prolapse, tricuspid valve prolapse, enlargement of the proximal pulmonary artery • Ocular: myopia, ectopia lentis, risk for retinal detachment • Skeletal: pectus excavatum/carinatum, tall stature, scoliosis, arachnodactyly, disproportionately long extremities for the length of the trunk Classical EDS is distinguished from Marfan syndrome by the presence of atrophic scars, skin, and joint hyperextensibility. PREP Pearls • Cardinal features of Ehlers-Danlos syndromes include joint hypermobility, skin hyperextensibility, and tissue fragility. • In addition to the minimal clinical criteria, the diagnosis of classical Ehlers-Danlos syndrome requires the identification of a pathogenic variant in one of the known associated genes (COL5A1, COL5A2, COL1A1). • Evaluation and management of classical Ehlers-Danlos syndrome includes assessment of clotting parameters, baseline 2D echocardiography, evaluation and therapy for hypotonia and gross motor delay, and management of pain and wound healing. ABP Content Specifications(s) • Recognize the clinical findings associated with Ehlers-Danlos syndrome

American Academy of Pediatrics

35

PREP® Self-Assessment

PREPSA 2023

Suggested Readings • Bowen JM, Sobey GJ, Burrows NP, et al. Ehlers-Danlos syndrome, classical type. Am J Med Genet C Semin Med Genet. 2017;175(1):27-39. doi:10.1002/ajmg.c.31548. • Dietz H. FBN1-related Marfan syndrome. GeneReviews. GeneReviews [Internet]. University of Washington; 2021. Accessed September 1, 2022. https://www.ncbi.nlm.nih.gov/books/NBK1335/. • Malfait F, Francomano C, Byers P, et al. The 2017 international classification of the Ehlers-Danlos syndromes. Am J Med Genet C Semin Med Genet. 2017;175(1):8-26. doi:10.1002/ajmg.c.31552. • Malfait F, Wenstrup R, De Paepe A. Classic Ehlers-Danlos syndrome. GeneReviews [Internet]. University of Washington; 2021. Accessed September 1, 2022. https://www.ncbi.nlm.nih.gov/books/NBK1244/. • Siegel DM, Marston B. Joint pain. In: McInerny TK, Adam HM, Campbell DE, DeWitt TG, Foy JM, Kamat DM, eds. American Academy of Pediatrics Textbook of Pediatric Care. American Academy of Pediatrics; 2021:chap 171. Accessed September 1, 2022. Pediatric Care Online .

American Academy of Pediatrics

36

PREP® Self-Assessment

PREPSA 2023

Question 11 A 7-year-old boy is seen for a health supervision visit after being lost to follow-up. His mother reports that he has received special education services since kindergarten and is in a “separate classroom.” She is unsure of his special education classification, but states that “he acts just like my 3-year-old.” The boy was delayed with toilet training, and this past year learned to dress himself. He only recently began identifying colors and does not know the alphabet. During the physical examination, the boy is eager for attention and shows the examiner an action figure he brought with him. He can provide his name and age, and speaks in 3-word sentences. He shakes his head “no” or shrugs when asked open-ended questions, and maintains eye contact. The remainder of the boy’s physical examination findings are normal. After the physical examination is completed, he gestures to request the stethoscope so that he can listen to his action figure. Of the following, this boy’s MOST likely diagnosis is A. autism spectrum disorder B. hearing impairment C. intellectual disability D. language disorder

American Academy of Pediatrics

37

PREP® Self-Assessment

PREPSA 2023

Correct Answer: C The boy in the vignette is demonstrating impairments in cognitive and adaptive skills, consistent with a diagnosis of intellectual disability. The boy’s behavioral and developmental history, along with the speech, language, and social skills he demonstrates during the visit, are at a 3-year-old level. The physician should confirm this diagnosis and its severity by reviewing the neuropsychological testing report from the school. Autism spectrum disorder is defined by social communication deficits including difficulties with non-verbal communication. Repetitive and restricted behaviors are also required for a diagnosis of autism. The boy in the vignette makes eye contact, uses gestures to support his communication, and demonstrates social reciprocity by showing and sharing objects. Additionally, by observation and parent report, he does not have repetitive or restricted behaviors or interests. The boy’s social and play skills are at the same level as his cognitive and adaptive skills, as expected in a child with intellectual disability. Although intellectual disability co-occurs in about 30% of individuals with autism spectrum disorder, those with autism have more prominent social communication deficits with social functioning that is lower than expected for their cognitive level. When assessing a child with language delays, including those with suspected autism spectrum disorder, hearing impairment must be considered; formal audiology evaluation should be completed. However, hearing impairment alone would not account for the cognitive and adaptive impairments exhibited by the boy in the vignette. Similarly, a diagnosis of language disorder does not explain the impairment in non-verbal and adaptive skills exhibited by this boy. PREP Pearls • Intellectual disability co-occurs in about 30% of individuals with autism spectrum disorder. Individuals with both autism spectrum disorder and intellectual disability have more prominent social communication deficits with social functioning that is lower than expected for their cognitive level. • Formal audiology evaluation is a crucial part of the initial evaluation for children with language delays or concerns for autism spectrum disorder. ABP Content Specifications(s) • Distinguish findings associated with autism spectrum disorder from those of an intellectual disability • Distinguish findings associated with autism spectrum disorder from those of profound hearing loss

American Academy of Pediatrics

38

PREP® Self-Assessment

PREPSA 2023

Suggested Readings • Hyman SL, Levy SE, Myers SM; Council on Children With Disabilities, Section on Developmental and Behavioral Pediatrics. Identification, evaluation, and management of children with autism spectrum disorder. Pediatrics. 2020;145(1):e20193447. doi:10.1542/peds.2019-3447. • Kryszak E, Mulick JA, Butter EM. Autism spectrum disorder. In: McInerny TK, Adam HM, Campbell DE, DeWitt TG, Foy JM, Kamat DM, eds. American Academy of Pediatrics Textbook of Pediatric Care. American Academy of Pediatrics; 2021:chap 221. Accessed September 1, 2022. Pediatric Care Online. • Long M, Register-Brown K. Autism spectrum disorder. Pediatr Rev. 2021;42(7):360-374. doi:10.1542/pir.2020-000547. • Phelps RA, Cohen WI. Intellectual disability. In: McInerny TK, Adam HM, Campbell DE, DeWitt TG, Foy JM, Kamat DM, eds. American Academy of Pediatrics Textbook of Pediatric Care. American Academy of Pediatrics; 2021:chap 278. Accessed September 1, 2022. Pediatric Care Online. • Purugganan O. Intellectual disabilities. Pediatr Rev. 2018;39(6):299-309. doi:10.1542/pir.2016-0116.

American Academy of Pediatrics

39

PREP® Self-Assessment

PREPSA 2023

Question 12 A 16-year-old adolescent boy is seen as a new patient to initiate care and discuss his attentiondeficit/hyperactivity disorder (ADHD). He takes amphetamine-dextroamphetamine 30 mg each morning and short-acting amphetaminedextroamphetamine 5 mg each afternoon. He was due for his annual health supervision visit and medication refill visit 4 months ago, but his mother recently had knee surgery and delayed bringing him in for his scheduled visit. The boy reports weekly marijuana use on a substance use screening tool. Of the following, this adolescent’s MOST likely source for nonmedical prescription medication use would be A. buying from a drug dealer B. his mother’s medication C. his own medication D. stealing from a health care professional

American Academy of Pediatrics

40

PREP® Self-Assessment

PREPSA 2023

Correct Answer: B The adolescent in the vignette is most likely to participate in nonmedical use of prescription medication obtained from his family member (ie, his mother). He is less likely to use his own medication for nonmedical reasons, steal the prescription or medication from a health care professional, or buy it from a drug dealer. Pediatricians have the important task of providing anticipatory guidance around safe medication use, storage, and disposal. According to the most recent National Survey on Drug Use and Health (2019), nonmedical use of prescription pain relievers and benzodiazepines has declined slightly among people aged 12 years and older (Item C12). Of those surveyed, 3.5% had used prescription pain medication for a nonmedical reason at least once in the past year. Of the 9.7 million people aged 12 years or older who used a prescription pain reliever in an inappropriate manner, over half of them reported obtaining the prescription medication from a friend or family member.

Approximately one-third inappropriately used a pain reliever medication that had been appropriately prescribed by their own health care professional. Of those individuals reporting nonmedical use of a pain reliever in the past year, 6.2% acquired the medication from a drug dealer. Prevention of nonmedical prescription drug use is an important part of adolescent health care. Physicians should counsel parents regarding the safe storage of all controlled substance prescription medications in a lock box or other secure location. These medications should be administered to the adolescent by a parent or guardian to ensure appropriate usage. Counseling around safe medication use, storage, and disposal should be provided for all family members (including grandparents and those outside the direct household). Health care professionals should be aware of their own community’s resources for controlled substance (eg, opioids) disposal such as pharmacy takeback days and at-home medication disposal options.

American Academy of Pediatrics

41

PREP® Self-Assessment

PREPSA 2023

PREP Pearls • Adolescents participating in nonmedical prescription drug use are most likely to obtain them from a friend or family • member. • Counseling around safe medication use, storage, and disposal for all family members is recommended. • Health care professionals should be knowledgeable about their local safe medication disposal options. ABP Content Specifications(s) • Recognize the risk of misuse of prescription medications Suggested Readings • Levy S, Bagley S. Substance use: initial approach in primary care. In: McInerny TK, Adam HM, Campbell DE, DeWitt TG, Foy JM, Kamat DM, eds. American Academy of Pediatrics Textbook of Pediatric Care. American Academy of Pediatrics; 2021:chap 239. AccessedSeptember 1, 2022.Pediatric Care Online . • Levy SJ, Williams JF; Committee on Substance Use and Prevention. Substance use screening, brief intervention, and referral to treatment. Pediatrics. 2016;138(1):e20161211. doi:10.1542/peds.2016-1211. • Substance Abuse and Mental Health Services Administration. (2020). Key substance use and mental health indicators in the United States: results from the 2019 National Survey on Drug Use and Health 2020. Accessed September 1, 2022. • https://www.samhsa.gov/data/sites/default/files/reports/rpt29393/2019NSDUHFFRPDF WHTML/2019NSDUHFFR090120.htm#topofpage.

American Academy of Pediatrics

42

PREP® Self-Assessment

PREPSA 2023

Question 13 A 4-year-old girl is evaluated in the emergency department for 6 days of bloody diarrhea. Her mother reports that her daughter appears tired and pale. The girl drank only 4 oz of fluids today and has not urinated for more than 24 hours. The girl’s 2 older sisters also have diarrhea, but their symptoms are improving. The family recently vacationed at a cabin on a lake in a rural area. They did not drink any unfiltered lake water but drank the tap water. The patient has a heart rate of 145 beats/min, a respiratory rate of 20 breaths/min, and blood pressure of 85/55 mm Hg. She is ill-appearing, and her capillary refill time is 4 seconds. The remainder of her physical examination findings are unremarkable. Of the following, the MOST likely cause of this girl’s illness is A. Escherichia coli B. Giardia duodenalis C. norovirus D. Yersinia enterocolitica

American Academy of Pediatrics

43

PREP® Self-Assessment

PREPSA 2023

Correct Answer: A The girl in the vignette has bloody diarrhea that most likely resulted from drinking contaminated well water, a common source of water in rural areas. The most likely cause of her illness is infection with Escherichia coli (possibly hemolytic uremic syndrome). E coli, including E coli 0157:H7 and other forms of Shiga toxin–producing E coli, are common well-water contaminants. Drinking water contaminated with Giardia duodenalis, norovirus, and Yersinia enterocolitica can also cause diarrhea but is less likely to produce bloody diarrhea. An estimated 15% to 20% of homes in the United States rely on private wells for their water supply. In most states, families are responsible for maintaining and testing their own water system. Well water should be tested at least annually for contaminants, including coliforms; if results are positive, specific testing for E coli should be performed. Other common bacterial contaminants include Salmonella, Yersinia, Shigella, and Campylobacter. Common viral contaminants include norovirus, sapovirus, rotavirus, and hepatitis A and E. Parasites, such as Giardia, can spread through water sources, and water filtration may not remove the spores. Routine testing for these parasites is not recommended unless there is an outbreak in the surrounding area or home. Chemicals, such as arsenic, fluoride, lead, nitrates, radon, and uranium, can also contaminate drinking water. Arsenic, radon, and uranium are carcinogenic. Arsenic is most commonly found in water in the Slate Belt (parts of the southeastern United States, Nebraska, and Alaska). Uranium is most commonly found in water in the Western mountain states (Arizona, Utah, New Mexico, Montana, Colorado, Idaho, Nevada, and Wyoming) and in granite outcroppings in the Eastern United States. Small amounts of fluoride in drinking water promote healthy tooth enamel, but toxic levels can be found in some private sources of drinking water (eg, well water). Lead can be found in private wells and public drinking water sources, typically as a result of leaching from lead-containing pipes. In 2016, the American Academy of Pediatrics published a policy statement on the prevention of childhood lead toxicity, which addressed the topic of lead exposure in schools from water fountains. Nitrates pose a significant risk to young infants fed formula mixed with contaminated water or homemade baby food made with vegetables contaminated with nitrate-containing fertilizer. Because of their increased ability to convert nitrates to nitrites, these infants are at risk of methemoglobinemia. During health supervision visits, practitioners should inquire about the family’s water supply, especially in rural areas, where private wells may be common. Families who depend on well water should be counseled to have their water tested at least once a year and more often when there are infants, pregnant women, or those with compromised immune systems drinking the water.

American Academy of Pediatrics

44

PREP® Self-Assessment

PREPSA 2023

PREP Pearls • Safe water sources should be discussed at health supervision visits. • Private well water sources should undergo annual testing for infectious or toxic contaminants. • Water from drinking fountains can be contaminated with lead; schools should test their water sources. ABP Content Specifications(s) • Know the contaminants potentially found in drinking water Suggested Readings • American Academy of Pediatrics. Drinking water from private wells and risks to children. Pediatrics. 2009;123(6):15991605. doi:10.1542/peds.2009-0751. • Council on Environmental Health. Prevention of childhood lead toxicity. Pediatrics. 2016;138(1):e20161493. doi:10.1542/peds.2016-1493. • US Environmental Protection Agency. Private drinking water wells. Accessed September 1, 2022. https://www.epa.gov/privatewells.

American Academy of Pediatrics

45

PREP® Self-Assessment

PREPSA 2023

Question 14 A 14-year-old adolescent girl is evaluated in the emergency department for fever of 3 days’ duration, sore throat, dysphagia, and a slightly muffled voice. She is otherwise healthy, fully immunized, and does not have a history of recurrent throat infections. Her family history is unremarkable. On physical examination, her temperature is 39°C, heart rate is 100 beats/min, respiratory rate is 18 breaths/min, and oxygen saturation is 99% in room air. She is in mild discomfort due to her throat pain but is speaking comfortably in complete sentences. Her left tonsil is 3+ enlarged and erythematous, her right tonsil is 1+, and her uvula is deviated to the right. She has enlargement and tenderness of her left cervical lymph nodes. The remainder of her physical examination findings are unremarkable. Of the following, the MOST appropriate next management step for this adolescent is A. drainage of the abscess B. intravenous antibiotics C. observation only D. tonsillectomy

American Academy of Pediatrics

46

PREP® Self-Assessment

PREPSA 2023

Correct Answer: A The adolescent in the vignette has a peritonsillar abscess (PTA); drainage of the abscess is the most appropriate first step in treatment. Intravenous antibiotics may be administered as the first step in treatment for individuals with certain complications resulting from PTAs (eg, septic thrombophlebitis). Observation alone is not appropriate management and may result in complications. Tonsillectomy for an acute infected PTA may be performed in certain individuals, such as those who would require general anesthesia for incision and drainage and those with a history of recurrent tonsil infections, but would not be the appropriate first-line treatment for this otherwise healthy adolescent. Peritonsillar abscesses occur when pus collects between the pharyngeal muscles and the palatine tonsil capsule. The most common inciting pathogens are Streptococcus and Fusobacterium species. Common clinical manifestations include fever, sore throat, dysphagia, muffled voice, asymmetric tonsils, and uvular deviation. The diagnosis of a PTA is usually made clinically. If the diagnosis is uncertain, intraoral ultrasonography or computed tomography of the neck with intravenous contrast may be performed to assess for the presence of a fluid collection or neck mass, taking into consideration the risks of sedation and radiation. Treatment of an uncomplicated PTA is usually drainage under local anesthesia with a subsequent 10-day course of oral antibiotics, usually a penicillin, cephalosporin, or clindamycin. After abscess culture and sensitivities are resulted, antibiotics can be directed at the causative organism. Some children and adolescents with PTA may require hospital admission, including those who experience complications. Indications for admission in PTA • The need for intravenous hydration due to poor oral intake • pain management • no reliable outpatient follow-up • management of complications after drainage such as severe bleeding or respiratory distress secondary to aspiration of abscess contents into the patient’s airway PREP Pearls • Peritonsillar abscesses typically present with unilateral tonsillar enlargement and uvular deviation. • Uncomplicated peritonsillar abscesses may be treated with drainage under local anesthesia and subsequent oral antibiotics.

American Academy of Pediatrics

47

PREP® Self-Assessment

PREPSA 2023

ABP Content Specifications(s) • Recognize the clinical findings associated with peritonsillar abscess • Plan the appropriate management of a peritonsillar abscess • Plan the appropriate diagnostic evaluation of a peritonsillar abscess, considering commonly associated pathogens Suggested Readings • Bochner RE, Gangar M, Belamarich PF. A clinical approach to tonsillitis, tonsillar hypertrophy, and peritonsillar and retropharyngeal abscesses. Pediatr Rev. 2017;38(2):81-92. doi:10.1542/pir.2016-0072. • Conrad C, Cornfield DN. Airway obstruction. In: McInerny TK, Adam HM, Campbell DE, DeWitt TG, Foy JM, Kamat DM, eds. American Academy of Pediatrics Textbook of Pediatric Care. American Academy of Pediatrics; 2021:chap 348. Accessed September 1, 2022. Pediatric Care Online . • Mitchell RB, Archer SM, Ishman SL, et al. Clinical practice guideline: tonsillectomy in children (update). Otolaryngol Head Neck Surg. 2019;160(1_suppl):S1-S42. doi:10.1177/0194599818801757.

American Academy of Pediatrics

48

PREP® Self-Assessment

PREPSA 2023

Question 15 A 16-year-old adolescent girl is seen in the emergency department for evaluation of chest pain, shortness of breath, and nausea of 1 day’s duration. She describes the pain as sharp, substernal, and constant. She feels nauseated but has not vomited, and has no interest in eating or drinking. The girl’s heart rate is 120 beats/min, respiratory rate is 40 breaths/min, blood pressure is 100/70 mm Hg, and oxygen saturation is 94% in room air. Her chest pain is not reproducible by palpation. Cardiac examination reveals a normal S1, normal S2 with a gallop rhythm, and no murmur. There are crackles heard bilaterally on auscultation of her lungs. Her liver is palpated 3 cm below the right costal margin. Her extremities are pale and cool with delayed capillary refill. A chest radiograph (Item Q15A) and electrocardiogram (Item Q15B) are obtained.

Of the following, the BEST next step in the adolescent’s diagnostic evaluation is A. cardiac catheterization B. cardiac magnetic resonance imaging C. transesophageal echocardiography D. transthoracic echocardiography

American Academy of Pediatrics

49

PREP® Self-Assessment

PREPSA 2023

Correct Answer: D The adolescent in the vignette has findings significant for tachycardia, a gallop rhythm, bilateral crackles, and hepatomegaly. Chest radiography demonstrates cardiomegaly and pulmonary edema, and low-voltages and abnormal ST segments and T waves are noted on electrocardiography. This constellation of signs and symptoms is consistent with myocarditis, and additional diagnostic testing is warranted. Of the response choices, the best next test would be transthoracic echocardiography. Transesophageal echocardiography is not recommended for the initial evaluation of myocarditis and is typically not needed in this clinical context. Cardiac magnetic resonance imaging (MRI) can play an important role in the diagnosis of myocarditis, but it would not be recommended for the initial evaluation. Cardiac catheterization may be warranted during the evaluation of heart failure, but it would not be appropriate at this stage of diagnosis. Myocarditis can present in several ways along a spectrum of severity. Individuals with fulminant myocarditis, the most severe form, are critically ill with poor perfusion, a gallop rhythm, and hepatomegaly. They may experience life-threatening arrhythmias and cardiovascular collapse. Due to the acute onset, the left ventricle may not have had an opportunity to dilate, so chest radiography may not show cardiomegaly nor will echocardiography show a dilated left ventricle, although the cardiac function will be poor. Signs and symptoms of heart failure in the nonfulminant form may include shortness of breath, dyspnea with exertion, nausea, vomiting, and decreased appetite (poor oral intake in infants). Chest radiography will demonstrate cardiomegaly and pulmonary edema. Echocardiography will show poor function and a dilated left ventricle. Dilated cardiomyopathy, as a result of myocarditis causing a “burned out” myocardium, presents with a gradual progression of heart failure signs and symptoms. The muscle becomes injured by infection and chronic inflammation, resulting in a thinned out and dilated left ventricle that functions poorly. In cases of myocarditis, chest radiography findings can range from normal to cardiomegaly and pulmonary edema, as seen in this girl’s imaging study. Echocardiography will show varying degrees of ventricular dysfunction and possibly a pericardial effusion and valvular dysfunction. Electrocardiography should be obtained on all patients with suspected myocarditis. Most affected individuals will be tachycardic and some will have arrhythmias (eg, tachyarrhythmias and heart block). Relatively low voltages are commonly seen, as are abnormal T waves. Rarely, there may be changes suggestive of ischemia with ST segment elevations (Item C15). A cardiac MRI may be used to look for evidence of myocardial inflammation and scarring. Laboratory studies that may prove helpful in the evaluation of myocarditis include troponin, brain-natriuretic peptide, end-organ function studies (eg, blood urea nitrogen, creatinine, transaminases), as well as viral polymerase chain reaction studies and serologies.

American Academy of Pediatrics

50

PREP® Self-Assessment

PREPSA 2023

PREP Pearls • Myocarditis can present along a spectrum of severity ranging from cardiogenic shock to a chronic picture with dilated cardiomyopathy. • Electrocardiography can be very helpful in the diagnosis of myocarditis, often demonstrating low voltage, ST- and Twave abnormalities, and arrhythmias. • Transthoracic echocardiography can be used to support the diagnosis of myocarditis demonstrating evidence of poor function with or without dilation of the left ventricle, depending on the chronicity of disease. MOCA-Peds Objective • Recognize the clinical manifestations, etiology, and diagnosis of myocarditis. ABP Content Specifications(s) • Plan an appropriate diagnostic evaluation of myocarditis Suggested Readings • Dasgupta S, Iannucci G, Mao C, Clabby M, Oster ME. Myocarditis in the pediatric population: a review. Congenit Heart Dis. 2019;14(5):868-877. doi:10.1111/chd.12835. • McCulloch MA, Gajarski RJ. Congenital and acquired heart disease. In: McInerny TK, Adam HM, Campbell DE, DeWitt TG, Foy JM, Kamat DM, eds. American Academy of Pediatrics Textbook of Pediatric Care. American Academy of Pediatrics; 2021:chap 234. Accessed September 1, 2022. Pediatric Care Online. • Putschoegel A, Auerbach S. Diagnosis, evaluation and treatment of myocarditis in children. Pediatr Clin North Am. 2020;67(5):855-874. doi:10.1016/j.pcl.2020.06.013. • Tunuguntla H, Jeewa A, Denfield SW. Acute myocarditis and pericarditis in children. Pediatr Rev. 2019;40(1):14-25. doi:10.1542/pir.2018-0044. American Academy of Pediatrics 51

PREP® Self-Assessment

PREPSA 2023

Question 16 A 13-year-old adolescent girl is seen in the office for evaluation of lower abdominal pain and hematochezia of 3 months’ duration. Her symptoms have progressed from mild lower abdominal pain and loose stools to severe, crampy lower abdominal pain with bloody, liquid stools 6 to 8 times per day. She has urgency and tenesmus, and passes nocturnal stools. Her family has noted pallor over the last few weeks, and she reports dizziness. The girl is otherwise healthy and does not take any medication. Her mother has rheumatoid arthritis and thyroid disease. The girl is pale and quiet. She is afebrile, with a heart rate of 150 beats/min, blood pressure of 110/65 mm Hg, weight of 38 kg (13th percentile for age), height of 146 cm (4th percentile for age), and body mass index of 18 kg/m2 (30th percentile for age). She has conjunctival pallor, dry mucous membranes, and a soft systolic murmur. Her abdomen is soft, tender to palpation diffusely without rebound or guarding, and grossly bloody stool is noted on the rectal examination. Laboratory evaluation in the office demonstrates a hemoglobin level of 7.8 g/dL (78 g/L) and positive stool occult blood test. Of the following, the BEST next step in the management of this adolescent is A. immediate referral to the local emergency department for evaluation and care B. oral iron supplementation with repeat hemoglobin test in 1 week C. referral to pediatric gastroenterology for appointment within 1 week D. stool infection studies with follow-up office appointment in 24 hours

American Academy of Pediatrics

52

PREP® Self-Assessment

PREPSA 2023

Correct Answer: A The adolescent in the vignette has significant anemia, evidence of ongoing lower gastrointestinal bleeding, and vital signs that are concerning for hypovolemia (tachycardia). She needs to be urgently evaluated and stabilized in the emergency department. The initial step in the evaluation of children with gastrointestinal bleeding (either through vomitus or stool) is the assessment of hemodynamic stability based on vital signs (tachycardia, hypotension, orthostatic changes), capillary refill, and mental status. If hemodynamic instability is noted, appropriate intravenous (IV) access and IV fluid will be necessary, with consideration of transfusions of packed red blood cells, platelets, and/or coagulation factors as clinically appropriate. Laboratory data, including hemoglobin level, platelet count, coagulation studies, liver function tests, albumin level, and type and cross-match (in preparation for potential blood transfusions), should be obtained. The presence of blood should be confirmed with a stool occult blood test, as stool or vomitus can appear red with the ingestion of certain foods, dyes, and medications. Urgent consultation with a pediatric gastroenterologist is indicated because children with refractory gastrointestinal bleeding and ongoing hemodynamic instability despite the administration of IV fluids and blood products may require emergent therapeutic endoscopy. Once hemodynamic stability is established, attention should be given to the cause of the gastrointestinal bleeding. Upper gastrointestinal (UGI) bleeding may manifest as hematemesis (bright red blood in vomitus), coffee-ground emesis, melena, or (with brisk UGI bleeding) hematochezia. Lower gastrointestinal (LGI) bleeding may present as melena or hematochezia. Common causes of UGI bleeding are as follows: • Coagulopathy • Esophageal varices • Esophagitis • Gastritis • Mallory-Weiss tear • Ulcer (gastric or duodenal, peptic or Helicobacter pylori–associated) Common causes of LGI bleeding include: • Arteriovenous malformation • Coagulopathy • Colitis (infectious, allergic, inflammatory bowel disease) • Fissure • Intussusception • Meckel diverticulum Polyp Evaluation to determine the source of the gastrointestinal bleeding should be tailored to the suspected causes. Endoscopy should be used to evaluate UGI bleeding to both determine the American Academy of Pediatrics

53

PREP® Self-Assessment

PREPSA 2023

cause of bleeding and potentially treat the underlying condition. Treatment methods include variceal banding, argon plasma coagulation, cautery, and epinephrine administration. The approach to determining the cause of LGI bleeding should be tailored by the presenting symptoms. A Meckel scan should be considered for painless LGI bleeding. A history supporting colitis (abdominal pain, urgency, tenesmus, and/or passing of nocturnal stools) should prompt an evaluation for infectious colitis. Colicky pain and vomiting in a young child would suggest intussusception, and ultrasonography with surgical consultation should be considered. Ongoing LGl bleeding would require further evaluation with endoscopy and colonoscopy. Assessing the small bowel with a nuclear medicine scan (using technetium Tc 99m-labeled red blood cells), angiography, and/or capsule endoscopy may be recommended when gastrointestinal bleeding continues and endoscopy and colonoscopy are unable to reveal the source. Starting oral iron supplementation with follow-up in 1 week or requesting pediatric gastroenterology consultation within 1 week are not appropriate steps for this adolescent with evidence of hypovolemia and anemia on physical examination. Although stool infection studies would be indicated, follow-up in 24 hours would not be appropriate for this adolescent. PREP Pearls • Initial assessment of a child with gastrointestinal bleeding includes evaluation of hemodynamic stability. • Children with gastrointestinal bleeding and hemodynamic instability need urgent intravenous access and intravenous fluid administration, and may require transfusion of blood products. • Upper gastrointestinal bleeding may manifest as hematemesis, melena, or hematochezia. Lower gastrointestinal bleeding may present as melena or hematochezia. ABP Content Specifications(s) • Plan the appropriate evaluation of blood in vomitus or stool, including in a patient who has hemodynamically significant blood loss

American Academy of Pediatrics

54

PREP® Self-Assessment

PREPSA 2023

Suggested Readings • Avner JR. Gastrointestinal hemorrhage. In: McInerny TK, Adam HM, Campbell DE, DeWitt TG, Foy JM, Kamat DM, eds. American Academy of Pediatrics Textbook of Pediatric Care. American Academy of Pediatrics; 2021:chap 155. Accessed September 1, 2022. Pediatric Care Online. • Neidich GA, Cole SR. Gastrointestinal bleeding. Pediatr Rev. 2016;35(6):243-254. doi:10.1542/pir.35-6-243. • Sahn B, Bitton S. Lower gastrointestinal bleeding in children. Gastrointest Endoscopy Clin North Am. 2016;26:75-98. doi:10.1016/j.giec.2015.08.007. • Turner D, Ruemmele FM, Orlanski-Meyer E, et al. Management of paediatric ulcerative colitis, part 2: acute severe colitis-an evidence-based consensus guideline from the European Crohn’s and Colitis Organization and the European Society of Paediatric Gastroenterology, Hepatology, and Nutrition. J Pediatr Gastrointest Nutr. 2018;67:292310. doi:10.1097/MPG.0000000000002036.

American Academy of Pediatrics

55

PREP® Self-Assessment

PREPSA 2023

Question 17 A 4-month-old, full-term, developmentally normal infant is evaluated in the outpatient office for paroxysmal episodes for the past 2 weeks. During the episodes, his body crunches up repeatedly, “as though he is doing baby sit-ups.” Initially, it occurred once or twice per day, usually in the evening, but it now occurs in clusters, at which point he becomes difficult to console. His mother suspects that he is uncomfortable and might have gastroesophageal reflux. She is concerned because the episodes are becoming more frequent. The infant’s vital signs are normal. His physical examination findings, including neurological, are normal. During the visit, the infant had a typical episode in which he stiffened, his arms extended out suddenly and he bent forward. This occurred in a short cluster during which he appeared uncomfortable and cried. Of the following, the BEST next step in this infant’s management is A. referral to a gastroenterologist for outpatient evaluation B. referral to a neurologist for outpatient evaluation C. reflux precautions and close follow-up in the pediatrician’s office D. urgent neurological evaluation through the emergency department

American Academy of Pediatrics

56

PREP® Self-Assessment

PREPSA 2023

Correct Answer: D The infant in the vignette’s episodes are suggestive of infantile spasms (IS), a condition requiring prompt diagnosis and treatment to optimize developmental outcome. Infantile spasms, the most common epilepsy syndrome in infancy, is clinically characterized by a triad of: • Epileptic spasms • Electroencephalogram background of hypsarrhythmia • Accompanying developmental plateau and regression When all 3 of these features are present the eponym, “West syndrome,” is used. The diagnosis requires a high index of suspicion and necessitates urgent evaluation with expedited electroencephalogram (EEG) and pediatric neurology consultation. Infantile spasms typically present during the first year after birth, with a peak age of onset between 3 and 7 months. Classification is divided into 2 etiologic categories: cryptogenic and symptomatic. Cryptogenic IS is diagnosed in 10% to 40% of affected infants, presenting in developmentally normal infants whose diagnostic evaluation does not identify an underlying cause. With prompt recognition and early initiation of treatment, this cohort of children is more likely to have a favorable developmental outcome. Symptomatic IS is diagnosed in 60% to 90% of affected infants; a greater percentage of infants are now recognized as symptomatic due to improved genetic diagnostic testing. Symptomatic IS can occur secondary to prenatal/perinatal causes including hypoxic-ischemic encephalopathy, cortical malformations, and genetic syndromes (classically, tuberous sclerosis complex and trisomy 21). Postnatal etiologies can include trauma and infection. Developmental outcome is highly dependent on the underlying etiology. Item C17A summarizes the etiologic classification of infantile spasms. Often, when symptoms of IS begin, parents bring their children to the pediatrician with concerns for colic or gastroesophageal reflux (GER). Classically, spasms involve symmetric contraction with flexion of the trunk, neck and arms lasting up to 5 seconds and occurring in clusters. However, there is a range of clinical appearance and severity of spasms, with some presenting as tonic eye rolling or neck flexion/nodding, which can be subtle and easily missed. Spasms may initially emerge while the infant is drowsy, either when falling asleep or waking from sleep, and can be accompanied by crying, grimacing, pallor, flushing, or nystagmus. Suspicion for infantile spasms should prompt urgent referral for evaluation and EEG rather than outpatient referral due to the impact early diagnosis and treatment can have on developmental outcome. While GER or colic can have a similar clinical appearance, IS should be considered and excluded first.

American Academy of Pediatrics

57

PREP® Self-Assessment

PREPSA 2023

Electroencephalography in cases of IS demonstrates a specific interictal background of hypsarrhythmia: a chaotic, disorganized, asynchronous, nonrhythmic, high-voltage spike and spike-and-slow wave pattern (Item C17B). While a routine EEG may capture hypsarrhythmia, a recording including wakefulness and sleep is helpful as hypsarrhythmia initially emerges during stage 2 and 3 of non-REM sleep; prolonged (24 hours) EEG monitoring allows exclusion of movements mimicking infantile spasms.

American Academy of Pediatrics

58

PREP® Self-Assessment

PREPSA 2023

Once the diagnosis of IS is confirmed, evaluation is directed at identifying an underlying cause. This process typically includes magnetic resonance imaging of the brain as well as genetic and metabolic testing, with an emphasis on treatable disorders with specific therapies. Treatment of IS includes administration of adrenocorticotropic hormone (ACTH) or vigabatrin initiated as soon as possible after diagnosis. Adrenocorticotropic hormone is preferred over vigabatrin for first-line management of cryptogenic spasms. Vigabatrin can be used first-line and is the preferred initial choice in children with tuberous sclerosis complex. Treatment considerations are summarized in Item C17C. Treatment effect is often reassessed both clinically and with repeat EEG monitoring to confirm resolution of the hypsarrhythmia pattern. Children with infantile spasms often require long-term, multidisciplinary monitoring for neurodevelopmental progress and subsequent development of other seizure types.

PREP Pearls • Infantile spasms, the most common epilepsy syndrome in infancy, are clinically characterized by a triad of: 1) epileptic spasms, 2) electroencephalogram background of hypsarrhythmia, and 3) accompanying developmental plateau and regression. • A high index of suspicion and urgent evaluation with expedited electroencephalography are necessary to confirm the diagnosis of infantile spasms and quickly initiate treatment. • Targeted evaluation for underlying causes of infantile spasms, focusing on treatable conditions, should occur concomitantly with treatment initiation and is tailored toward the individual patient. Long-term developmental outcome is impacted by time to treatment and underlying etiology. MOCA-Peds Objective • Recognize the clinical features of rheumatic fever. ABP Content Specifications(s) • Understand the prognosis of infantile spasms • Recognize the clinical findings associated with infantile spasms American Academy of Pediatrics

59

PREP® Self-Assessment

PREPSA 2023

Suggested Readings • Fine A, Wirrell E. Seizures in children. Pediatr Rev. 2020;41(7):321-347. doi:10.1542/pir.2019-0134. • Go CY, Mackay MT, Weiss SK, et al. Evidence based guideline update: medical treatment of infantile spasms. Report of the Guideline Development Subcommittee of the American Academy of Neurology and the Practice Committee of the Child Neurology Society. Neurology. 2012;78(24):1974-1980. doi:10.1212/WNL.0b013e318259e2cf. • McBride M. Seizure disorders. In: McInerny TK, Adam HM, Campbell DE, DeWitt TG, Foy JM, Kamat DM, eds. American Academy of Pediatrics Textbook of Pediatric Care. American Academy of Pediatrics; 2021:chap 327. Accessed September 1, 2022. Pediatric Care Online. • Whelass JW, Gibson PA, Rosbeck KL, et al. Infantile spasms (West syndrome): update and resources for pediatricians and providers to share with parents. BMC Pediatr. 2012;12:108. doi:10.1186/1471-2431-12-108.

American Academy of Pediatrics

60

PREP® Self-Assessment

PREPSA 2023

Question 18 A 5-year-old, previously healthy boy is seen in the clinic for evaluation of testicular pain and fever that started this morning. He had a cough and runny nose 2 weeks ago, which have resolved. He has no other symptoms and no history of trauma. On physical examination, the boy has a temperature of 38.5°C, heart rate of 117 beats/min, respiratory rate of 22 breaths/min, and oxygen saturation of 98% in room air. Both testicles are palpated in the scrotum and the cremasteric reflex is intact. The testicles appear swollen, with no abnormal coloring or masses palpated. Ultrasonography with Doppler shows normal blood flow to both testicles. Of the following, this boy’s MOST likely diagnosis is A. inguinal hernia B. postinfectious orchitis C. testicular torsion D. torsion of the appendix testis

American Academy of Pediatrics

61

PREP® Self-Assessment

PREPSA 2023

Correct Answer: B The boy in the vignette has fever, painful testicular swelling, and a normal cremasteric reflex. Of the response choices, these signs and symptoms are most consistent with orchitis. In this age group, orchitis is most commonly seen after viral infections. An inguinal hernia can cause scrotal pain and swelling but would be palpated during the genitourinary examination. Testicular torsion, a surgical emergency, is an unlikely diagnosis for the boy in the vignette given his normal blood flow on Doppler ultrasonography and normal cremasteric reflex. In the case of testicular torsion, the normal cremasteric reflex (retraction of the testis in response to touch on the upper thigh) is usually absent. Torsion of the appendix testis is also unlikely given the normal findings on ultrasonography and absence of a “blue dot sign” on physical examination. Orchitis (inflammation of the testis) and epididymitis (inflammation of the epididymis) have infectious or inflammatory etiologies. Symptoms and signs of orchitis and epididymitis include testicular swelling and tenderness, dysuria, urinary frequency and urgency, and often systemic symptoms (eg, fever). The etiology of orchitis and epididymitis varies by age. Children ages 2 to 13 years often have a postinfectious cause (most commonly Mycoplasma, enterovirus, or adenovirus). Vasculitis (eg, Henoch-Schönlein purpura) is another common cause of orchitis and epididymitis in this age group. Older children and adults are more likely to have an infectious etiology with organisms that cause urinary tract or sexually transmitted infections. A urinalysis may show the presence of nitrites and/or leukocyte esterase because the infection can start in the urine and ascend into the epididymis and testes. It is important to differentiate testicular pain caused by orchitis and epididymitis from testicular torsion, which is an emergency. Testicular torsion can be difficult to diagnose based on physical examination findings alone. Therefore, ultrasonography with Doppler should be obtained for all children and adolescents with testicular pain. Ultrasonography with Doppler is not 100% specific; urgent surgical or urologic consultation should be obtained when there is concern for testicular torsion. PREP Pearls • All children and adolescents with testicular pain should have ultrasonography with Doppler performed to evaluate for testicular torsion. • Symptoms and signs of orchitis and epididymitis typically include testicular swelling and tenderness, dysuria, urinary frequency and urgency, and often systemic symptoms. ABP Content Specifications(s) • Recognize the clinical findings associated with orchitis • Identify common causes of orchitis

American Academy of Pediatrics

62

PREP® Self-Assessment

PREPSA 2023

Suggested Readings • Palmer LS. Scrotal swelling and pain. In: McInerny TK, Adam HM, Campbell DE, DeWitt TG, Foy JM, Kamat DM, eds. American Academy of Pediatrics Textbook of Pediatric Care. American Academy of Pediatrics; 2021:chap 190. Accessed September 1, 2022. Pediatric Care Online. • Sexually Transmitted Infections Treatment Guidelines, 2021. Epididymitis. Centers for Disease Control and Prevention. • Updated July 22, 2021. Accessed September 1, 2022. https://www.cdc.gov/std/treatmentguidelines/epididymitis.htm. • Stewart A, Ubee SS, Davies H. Epididymo-orchitis. BMJ. 2011;342:d1543. doi:10.1136/bmj.d1543. • Trojian TH, Lishnak TS, Heiman D. Epididymitis and orchitis: an overview. Am Fam Physician. 2009;79(7):583-587. https://pubmed.ncbi.nlm.nih.gov/19378875/. • Wu WJ, Gitlin JS. The male genital system. Pediatr Rev. 2020;41(3):101-111. doi:10.1542/pir.2017-0316.

American Academy of Pediatrics

63

PREP® Self-Assessment

PREPSA 2023

Question 19 A 9-month-old girl is seen for follow-up 3 weeks after hospital admission for a febrile Escherichia coli urinary tract infection. She received intravenous ceftriaxone in the hospital and subsequently completed treatment with oral cefdinir. The girl is currently asymptomatic, feeding well, and having regular bowel movements. At 6 months of age she had a febrile urinary tract infection. She is alert and active, and has vital signs that are normal for age. Her physical examination findings, including those of the genitourinary examination, are unremarkable. Renal ultrasonography performed in the hospital was normal. A voiding cystourethrogram showed bilateral grade III vesicoureteral reflux. Of the following, the MOST appropriate next management step for this infant is A. polyethylene glycol treatment, orally B. repeat voiding cystourethrogram in 3 months C. trimethoprim-sulfamethoxazole prophylaxis, orally D. ureteral reimplantation surgery

American Academy of Pediatrics

64

PREP® Self-Assessment

PREPSA 2023

Correct Answer: C The infant in the vignette has recurrent febrile urinary tract infection (UTI) and a voiding cystourethrogram (VCUG) that shows bilateral vesicoureteral reflux (VUR). The most appropriate next management step for this girl is trimethoprim-sulfamethoxazole prophylaxis to prevent recurrent UTI. Vesicoureteral reflux is the retrograde passage of urine from the bladder to the upper urinary tract. Vesicoureteral reflux is divided into primary and secondary types. Primary VUR occurs due to a congenitally short segment of ureter within the bladder wall, which results in incomplete closure of the ureterovesical junction during bladder contraction. Secondary VUR occurs as a result of high bladder pressure associated with conditions such as posterior urethral valves, bladder-bowel dysfunction, and neurogenic bladder. Asymptomatic vesicoureteral reflux may be diagnosed during evaluation of a neonate with a history of antenatal hydronephrosis or screening in children with a family history of VUR. Vesicoureteral reflux may be diagnosed during the evaluation of a febrile UTI. In toilet-trained children, VUR may present with bladder-bowel dysfunction (eg, urinary frequency and urgency, day time enuresis, encopresis and constipation). Less commonly, VUR may present with hypertension, renal insufficiency, or incidental detection of a small and scarred kidney on ultrasonography or computed tomography. Voiding cystourethrogram is the test of choice to confirm the presence of VUR and grade its severity. Indications to perform a VCUG in neonates with a history of antenatal hydronephrosis include presence of moderate to severe hydronephrosis or ureteral dilation on the postnatal ultrasonography. A VCUG is also indicated in a child with a first febrile UTI with abnormal renal ultrasonography, or a child with recurrent febrile UTI. Vesicoureteral reflux is categorized as low grade (I-III) and high grade (IV-V). The International Reflux study group classifies VUR into 5 grades as shown: • • • • •

Grade I: Reflux into the middle of the ureter without dilation Grade II: Reflux into the ureter and collecting system without dilation Grade III: Reflux into the ureter and collecting system with mild dilation and mild blunting of calices Grade IV: Reflux into the ureter and collecting system with gross dilation and severe blunting of calices Grade V: Massive reflux with gross dilation of ureter and collecting system, blunting of all calices, and tortuosity of the entire collecting system

Spontaneous resolution, without any surgical intervention, occurs in most cases of VUR. The likelihood of resolution is higher in low-grade as compared to high-grade VUR, unilateral as compared to bilateral VUR, and in those patients presenting with asymptomatic antenatal hydronephrosis compared to those presenting with febrile UTI. The medical management of VUR includes watchful waiting, addressing any bladder-bowel dysfunction, and starting American Academy of Pediatrics

65

PREP® Self-Assessment

PREPSA 2023

antibiotic prophylaxis (if indicated). Antibiotic prophylaxis is indicated for non–toilet-trained children, those with low-grade VUR with recurrent UTI, and those with high-grade VUR. Medications commonly used for UTI prophylaxis include trimethoprim-sulfamethoxazole, nitrofurantoin, or amoxicillin. Surgical correction (ureteral reimplantation) is recommended for patients who have recurrent breakthrough infections on antibiotic prophylaxis or persistent high grade VUR with significant risk of renal scarring. Polyethylene glycol treatment might be considered for a child with secondary VUR associated with chronic constipation. The child in the vignette has no history of constipation, therefore polyethylene glycol treatment would not be appropriate for her. A repeat VCUG to assess for resolution or persistence of VUR is generally recommended after 9 to 15 months, not after 3 months. The girl in the vignette does not meet criteria for surgical correction as she has bilateral grade III VUR (low grade) and has not had a breakthrough UTI with the use of antibiotics. PREP Pearls • Vesicoureteral reflux may be diagnosed during the evaluation of a febrile urinary tract infection, antenatally diagnosed hydronephrosis, or screening in children with family history. • Spontaneous resolution of vesicoureteral reflux is common when it is low grade, unilateral, and in those with asymptomatic antenatally diagnosed hydronephrosis. • Antibiotic prophylaxis is indicated for vesicoureteral reflux in non–toilet-trained children, those with low-grade vesicoureteral reflux with recurrent urinary tract infection, and those with high-grade vesicoureteral reflux. ABP Content Specifications(s) • Recognize the clinical findings associated with vesicoureteral reflux • Understand the natural history of vesicoureteral reflux Suggested Readings • Balighian E, Burke M. Urinary tract infections in children. Pediatr Rev. 2018;39(1):3-12. doi:10.1542/pir.2017-0007. • Mattoo TK, Shaikh N, Nelson CP. Contemporary management of urinary tract infection in children. Pediatrics. 2021;147(2):e2020012138. doi:10.1542/peds.2020-012138. • Nguyen HT. Obstructive uropathy and vesicoureteral reflux. In: McInerny TK, Adam HM, Campbell DE, DeWitt TG, Foy JM, Kamat DM, eds. American Academy of Pediatrics Textbook of Pediatric Care. American Academy of Pediatrics; 2021:chap 299. Accessed September 1, 2021. Pediatric Care Online.

American Academy of Pediatrics

66

PREP® Self-Assessment

PREPSA 2023

Question 20 A 12-year-old boy is seen in the office for evaluation of bilateral anterior knee pain. His pain began about 3 months ago during wrestling practice. There was no acute injury. He reports no swelling, locking, or instability. The pain is worse with kneeling, running, and jumping. There is tenderness over the inferior aspect of both patellae. He also has pain with resisted knee extension. The remainder of the boy’s physical examination findings are unremarkable. Of the following, the MOST likely cause of this boy’s pain is A. Osgood-Schlatter disease B. patellar tendinopathy C. prepatellar bursitis D. Sinding-Larsen-Johansson syndrome

American Academy of Pediatrics

67

PREP® Self-Assessment

PREPSA 2023

Correct Answer: D The most likely diagnosis for the boy in the vignette is Sinding-Larsen-Johansson syndrome (SLJ) or inferior patellar pole apophysitis. An apophysis is a bony ossification center adjacent to a minor growth plate. Apophyses are found at sites where tendons attach to bone. Contraction of a muscle causes the tendon to pull on the apophysis. The physis, the growth area adjacent to the apophysis, is made of soft bone that has not yet calcified and is especially vulnerable to injury. With repetitive activity, traction on the apophysis can lead to pain. Overuse of the quadriceps muscles and direct pressure on the bottom of the patella (eg, kneeling) cause irritation of the apophysis, known as apophysitis. Sinding-Larsen-Johansson syndrome typically occurs in children between the ages of 9 and 13 years. Because they experience rapid prepubertal growth later than girls, boys tend to have SLJ at the older end of this age range, while girls typically have symptoms around age 9 or 10 years. The boy in the vignette has classic history and physical examination findings for SLJ: insidious onset of pain, bilateral symptoms, tenderness over the inferior pole of the patella, and pain with resisted quadriceps contraction. About half of affected children have unilateral symptoms. Some children recall an acute episode that provoked their pain, such as a fall directly onto the knee(s). While clinicians can make the diagnosis of SLJ based on history and physical examination findings, radiographs often show an irregular pattern or a slight separation at the inferior patella apophysis. Item C20A shows a prominent apophysis at the inferior pole of the patella. Use of a patellar strap over the patellar tendon (Item C20B) can act like a pulley mechanism and relieve some of the tension on the apophysis. Strengthening and stretching exercises may help some children, but there are no studies to show that exercises are efficacious. The application of ice and the use of topical or oral nonsteroidal antiinflammatory medications (NSAIDs) may be helpful for relief of symptoms.

American Academy of Pediatrics

68

PREP® Self-Assessment

PREPSA 2023

Osgood-Schlatter disease (OSD) is another type of apophysitis. Children with OSD have pain and tenderness at the tibial tubercle, the site where the patellar tendon attaches. The clinical history and treatment of OSD and SLJ are very similar. Patellar tendinopathy involves breakdown and fraying of the patellar tendon. Tendinopathy is more common in adults, although adolescents who participate in jumping and landing sports experience this condition as well. Young athletes with patellar tendinopathy may have symptoms similar to those seen with SLJ and OSD. However, adolescents with patellar tendinopathy have tenderness over the tendon, rather than the bone. Historically, clinicians used the term tendinitis to describe tendon irritation. Currently, the term tendinopathy is preferred as this condition does not have an inflammatory cause. With repeated pressure or trauma to the front of the knee, the prepatellar bursa can become irritated and swollen, a condition known as prepatellar bursitis. Prepatellar bursitis is more common in adults but can affect adolescents and rarely, younger children. Because wrestlers often kneel and land forcefully on their knees during practice and competition, they are at increased risk of prepatellar bursitis. Affected individuals present with localized swelling over the patella. The presence of redness and warmth over the front of the knee should prompt evaluation for infectious bursitis which can be either primary, or secondary, following traumatic bursitis. PREP Pearls • Apophysitis occurs because the physis, the growth area adjacent to the apophysis, is made of soft bone and is especially vulnerable to injury. • Sinding-Larsen-Johansson syndrome is an apophysitis of the inferior patellar pole apophysis. Osgood-Schlatter disease is an apophysitis of the tibial tubercle. • Tendinopathy and bursitis, while frequently seen in adults, are uncommon causes of knee pain in children. ABP Content Specifications(s) • Recognize the clinical findings associated with sports-related prepatellar bursitis, and manage appropriately Suggested Readings • Kannikeswaran N, Suresh S. Sports musculoskeletal injuries. In: McInerny TK, Adam HM, Campbell DE, DeWitt TG, Foy JM, Kamat DM, eds. American Academy of Pediatrics Textbook of Pediatric Care. American Academy of Pediatrics; 2021:chap 334. Accessed September 1, 2022. Pediatric Care Online. • Kiningham R, Monseau A. Caring for wrestlers. Curr Sports Med Rep. 2015;14(5):404412. doi:10.1249/JSR.0000000000000193. • Overuse injuries. In: Sarwark JF, Labella CR, eds. Orthopaedics and Sports Medicine: A Quick Reference Guide. 3rd ed. American Academy of Pediatrics. 2021:311-342. • Wolf M. Knee pain in children: part I: evaluation. Pediatr Rev. 2016;37(1):18-23; quiz 24, 47. doi:10.1542/pir.2015-0040 American Academy of Pediatrics 69

PREP® Self-Assessment

PREPSA 2023

Question 21 A 20-month-old, previously healthy girl is brought to the urgent care center with a 1-day history of left otalgia and increased fussiness. She has had rhinorrhea and cough for the past week, and developed a fever of 38.8°C 6 hours ago that resolved after 1 dose of acetaminophen. There has been no otorrhea. She is drinking, eating, stooling, and voiding well. She has no past history of otitis media. On physical examination, she is afebrile and has clear rhinorrhea in both nares and 2 tender, mobile, soft, 1-cm posterior cervical lymph nodes. The appearance of her left tympanic membrane is shown in Item Q21. The remainder of her physical examination findings are normal.

Of the following, the BEST next step in this girl’s management is A. high-dose amoxicillin for 5 days B. high-dose amoxicillin for 7 days C. high-dose amoxicillin for 10 days D. observation for 48 to 72 hours

American Academy of Pediatrics

70

PREP® Self-Assessment

PREPSA 2023

Correct Answer: B The girl in the vignette has acute otitis media (AOM), which is an acute bacterial infection of the middle ear with fluid, otherwise known as suppurative otitis media. According to the American Academy of Pediatrics Acute Otitis Media Clinical Practice Guidelines, the best treatment for children younger than 24 months of age without severe signs or symptoms is observation for 48 to 72 hours with analgesic administration as needed followed by an antibiotic prescription if symptoms worsen or persist over that time frame. The diagnostic criteria for AOM include: • Recent onset of ear pain (100 milliseconds increases the risk for seizures; prolongation >160 milliseconds increases the risk for complex arrhythmias. Death is usually a result of ventricular arrhythmias, which may be refractory to defibrillation. The focus of treatment for TCA overdose is stabilization of the cardiopulmonary system, which includes airway protection and fluid resuscitation. Seizures should be treated initially with a benzodiazepine. An electrocardiogram should be obtained immediately when there is suspected or confirmed ingestion of a TCA. Administration of sodium bicarbonate should be initiated in any child with TCA ingestion who is hemodynamically unstable, acidotic, hypotensive, exhibiting seizure activity, or has a QRS prolongation of more than 100 milliseconds. It is administered intravenously as a rapid IV bolus of 1 to 2 mEq/kg followed by a continuous infusion. The goal of sodium bicarbonate therapy is to achieve and maintain a serum pH of 7.5 and narrow the QRS complex. PREP Pearls • An electrocardiogram should be obtained immediately in a child with suspected or confirmed ingestion of a tricyclic antidepressant. • Sodium bicarbonate is the mainstay of treatment in children with tricyclic antidepressant toxicity and should be initiated in any child who is hemodynamically unstable, acidotic, hypotensive, or exhibiting seizure activity or a QRS interval prolongation of more than 100 milliseconds. • The goal of sodium bicarbonate therapy for tricyclic antidepressant–induced QRS interval prolongation is to narrow the QRS complex and achieve and maintain a serum pH of 7.5. ABP Content Specifications(s) • Recognize the signs and symptoms of tricyclic antidepressant toxicity, and provide appropriate initial management • Recognize the signs and symptoms of ingestion of an anticholinergic drug, and manage appropriately Suggested Readings • Fine JS, Auerbach MA, Ching KU, Fullerton KT, Weinberg ER. Poisoning. In: McInerny TK, Adam HM, Campbell DE, DeWitt TG, Foy JM, Kamat DM, eds. American Academy of Pediatrics Textbook of Pediatric Care. American Academy of Pediatrics;2021:chap 369. Accessed September 1, 2022. Pediatric Care Online. • Toce MS, Burns MM. The poisoned pediatric patient. Pediatr Rev. 2017;38(5):207-220. doi:10.1542/pir.2016-0130.

American Academy of Pediatrics

435

PREP® Self-Assessment

PREPSA 2023

Question 129 A 3-year-old girl in the pediatric intensive care unit has acute respiratory failure from respiratory syncytial virus bronchiolitis. She is in marked respiratory distress, with a respiratory rate of 60 breaths/min and oxygen saturation of 73% in room air. She is somnolent and lethargic. Auscultation of the lungs demonstrates diffuse, coarse rales, wheezes, and rhonchi. The decision is made to perform rapid-sequence endotracheal intubation, after which her oxygen saturation is 98% on an FiO2 of 45%. Approximately 20 minutes after initiation of mechanical ventilation, the girl has an acute oxygen desaturation to 75% which does not improve when 100% FiO2 is provided by the ventilator. Visual inspection of her chest reveals equal but restricted chest rise, and auscultation reveals diminished breath sounds in all lung fields. Of the following, the BEST next step in this girl’s management is A. chest radiography B. manual ventilation with a resuscitation bag C. needle thoracostomy of the chest D. suctioning of the endotracheal tube

American Academy of Pediatrics

436

PREP® Self-Assessment

PREPSA 2023

Correct Answer: B The girl in the vignette has acute respiratory failure requiring intubation and mechanical ventilation. Given her ongoing hypoxia despite the provision of 100% FiO2 by the ventilator, equal but diminished breath sounds, and equal chest rise, the best next step in management is to provide manual ventilation with a resuscitation bag via the endotracheal tube to both assess and manage the cause of ventilation failure. Children with acute lung disease may progress to respiratory failure requiring intubation and mechanical ventilation due to: (1) failure of oxygenation and/or (2) failure of ventilation. Failure of oxygenation or ventilation can occur as a result of new-onset parenchymal lung disease, cardiac dysfunction, neurologic abnormalities, or multi-organ system failure. Children may also require intubation for airway protection, particularly when an acute underlying neurologic impairment renders them unable to protect their airway from aspiration of oral secretions (eg, acute head trauma or toxic ingestion). Endotracheal intubation should only be performed by those skilled in the procedure. Although rapid sequence endotracheal intubation can mitigate some of the complications, even in the most skilled hands, there are risks and complications that require vigilance. After successful placement of an endotracheal tube, complications can arise, as in the case of the girl in the vignette. A common mnemonic used to evaluate and manage a successfully intubated patient who subsequently develops hypoxemia is DOPE, which stands for: • Displacement/Dislodgement • Obstruction • Pneumothorax • Equipment failure

Each of these issues can be life-threatening if not addressed, but the first step in evaluating and managing a patient who is acutely hypoxic on the ventilator is to provide manual ventilation with a resuscitation bag. Displacement/dislodgement can occur after successful placement of an endotracheal tube in the trachea. The tube may migrate into one of the mainstem airways, or may become dislodged out of the trachea into the supraglottic space. Displacement can occur with patient movement or during the securement or re-securing of an endotracheal tube. Continuous end-tidal capnography can yield clues to the location of an endotracheal tube and can help verify that dislodgement has not occurred. The endotracheal tube can become obstructed with inspissated secretions that obstruct airflow down the tube. High airway resistance may be felt by the physician during manual ventilation with a flow-inflating resuscitation device. Suctioning the endotracheal tube can relieve the obstruction and improve airflow; this approach is often needed immediately after intubation in patients with copious pulmonary secretions. American Academy of Pediatrics

437

PREP® Self-Assessment

PREPSA 2023

Pneumothorax occurs when air leaks from ruptured alveoli into the pleural space leading to accumulation of air and compression or collapse of the affected lung. When the trapped air cannot escape, pressure and compression can lead to decreased venous return and cardiovascular collapse (eg, tension pneumothorax). In such cases, needle decompression is required to relieve the tension and a thoracostomy tube is often required thereafter. Equipment failure can occur when oxygen supply to the ventilator becomes disrupted, or when the ventilator itself malfunctions in some way such that the patient is not adequately supported. Although rare, equipment failure can be a cause of acute hypoxia after successful intubation. Although chest radiography could be helpful in diagnosing the cause of acute hypoxia, the time required would delay necessary and immediate intervention. Pneumothoraces tend to be unilateral with findings of absent or diminished breath sounds and decreased chest rise on the affected side only. The child in the vignette had bilateral findings, thus, needle decompression is not the preferred response. Finally, suctioning of the endotracheal tube can alleviate mucus or secretion-related endotracheal tube obstruction. However, given the ongoing significant oxygen desaturations, manual ventilation with a resuscitation bag is indicated before suctioning to attempt to restore normal oxygenation quickly. PREP Pearls • When acute hypoxia occurs in mechanically ventilated children, manual ventilation with a resuscitation bag should be performed with 100% FiO2 until the underlying cause of hypoxia can be addressed and corrected. • Endotracheal intubation may be required for children with respiratory failure, impending respiratory arrest, or those with neurologic impairment or severely altered mental status where airway protective reflexes cannot be maintained. • A common mnemonic used to evaluate and manage a successfully intubated patient who subsequently develops hypoxemia is DOPE: Displacement/Dislodgement of the endotracheal tube, Obstruction of the endotracheal tube, Pneumothorax, and Equipment failure. ABP Content Specifications(s) • Understand the potential complications associated with endotracheal intubation Suggested Readings • Easley RB, Segeleon JE, Haun SE, Tobias JD. Prospective study of airway management of children requiring endotracheal intubation before admission to a pediatric intensive care unit. Crit Care Med. 2000;28(6):2058-2063. doi:10.1097/00003246-20000600000065. • Hatch LD, Grubb PH, Lea AS, et al. Endotracheal intubation in neonates: a prospective study of adverse safety events in 162 infants. J Pediatr. 2016;168:62-66.e6. doi:10.1016/j.jpeds.2015.09.077. • Page NE, Giehl M, Luke S. Intubation complications in the critically ill child. AACN Clin Issues. 1998;9(1):25-35. American Academy of Pediatrics

438

PREP® Self-Assessment

PREPSA 2023

Question 130 A 10-year-old girl is seen for a health supervision visit. The girl recently completed a formal psychoeducational evaluation through her school because of long-standing learning difficulties and was diagnosed with dyslexia. The girl starts to cry. She admits that she is worried about no longer being in the same class as her friends and that the other students will “know I’m different because I have to go to the other classroom.” The school has a family meeting planned to discuss how special education services will be implemented. The girl’s mother received special education services for dyslexia when she was in school, which prevented her from attending her favorite class, which was art. She began investigating private school options for her daughter because she is worried that she will not have any input into determining how educational services will be provided. Of the following, the BEST next step in counseling this girl’s mother is to A. encourage her to continue her search for a private school so that she can be involved in decision-making B. have the mother request that special education instruction take place during the girl’s physical education class time C. recommend that the girl be in a regular education setting with in-class supports and small group instruction D. recommend that the girl be in a self-contained classroom for her entire school day

American Academy of Pediatrics

439

PREP® Self-Assessment

PREPSA 2023

Correct Answers: C In 1975 the Individuals with Disabilities Education Act (IDEA) was passed through Congress, which mandates that a free and appropriate public education be provided and that supports be implemented in the least restrictive environment allowing children with disabilities to be educated with children without disabilities as much as possible. For the girl in the vignette, inclass supports along with small group instruction would be considered the least restrictive educational environment. Pull-out instruction (having the girl leave her classroom periodically) may be considered if the girl does not meet the goals of her individualized education plan within the least restrictive setting. It is important for parents and caregivers to be involved in the educational planning process through shared decision-making. Parents should be encouraged to provide input into the implementation of special education services. Having the girl attend a private school does not guarantee that her mother would have more involvement in decision-making. Additionally, children placed in a private school by their parents do not have the same entitlement to special education services that they would receive by attending public school. A local or state agency may place a child in private school who was previously attending public school if the public school is unable to provide a free and appropriate education. Placement of the girl in a self-contained classroom would be the most restrictive environment and would not be an appropriate initial educational setting. For a child with more severe and global disabilities, a self-contained classroom may be an appropriate setting. However, opportunities for “main-streaming” during electives such as music, physical education, and recess should be encouraged to provide exposure to typical peers. The girl should not be removed from physical education as physical activity may help support self-esteem, improve mood, and provide additional opportunity for peer interaction. Pediatricians play an important role in not only identifying children in need of special education services, but in the implementation and longitudinal monitoring of educational interventions. Pediatricians should encourage parents and caregivers to take an active role in their children’s education plans and provide education on their right to a free and appropriate public education in the least restrictive environment. PREP Pearls • Children with disabilities must be educated in the least restrictive environment and, as much as possible, with children without disabilities. • All children are entitled to a free and appropriate public education with the school providing enough support for the child to progress. • Parents are key stakeholders in the design and implementation of special education services. ABP Content Specifications(s) • Recognize appropriate educational settings for patients with learning disabilities, and the various strategies utilized in those settings to circumvent weaknesses American Academy of Pediatrics

440

PREP® Self-Assessment

PREPSA 2023

Suggested Readings • Council on Early Childhood; Council on School Health. The pediatrician’s role in optimizing school readiness. Pediatrics. 2016;138(3):e20162293. doi:10.1542/peds.20162293. • Frankowski BL. Learning difficulty. In: McInerny TK, Adam HM, Campbell DE, DeWitt TG, Foy JM, Kamat DM, eds. American Academy of Pediatrics Textbook of Pediatric Care. American Academy of Pediatrics; 2021:chap 172. Accessed September 1, 2022. Pediatric Care Online. • Lipkin PH, Okamoto J; Council on Children with Disabilities; Council on School Health. The Individuals with Disabilities Education Act (IDEA) for children with special educational needs. Pediatrics. 2015;136(6):e1650-e1662. doi:10.1542/peds.2015-3409. • Rey-Casserly C, McGuinn L, Lavin A; Committee on Psychosocial Aspects of Child and Family Health,section on • Developmental and Behavioral Pediatrics. School-aged children who are not progressing academically: considerations for pediatricians. Pediatrics. 2019;144(4):e20192520. doi:10.1542/peds.2019-2520.

American Academy of Pediatrics

441

PREP® Self-Assessment

PREPSA 2023

Question 131 A 16-year-old adolescent boy is evaluated for left-sided neck swelling and pain of 5 days’ duration, headache, abdominal pain, and fever. No one else is sick at home and there has been no recent travel. The family has 1 chihuahua and 3 kittens. On physical examination, the boy is well appearing, with a temperature of 38.3°C. He has a tender neck mass on the left with overlying skin redness (Item Q131A). Neck ultrasonography shows a 3.3 × 2.1–cm suppurative lymph node in the neck left midjugular region (Item Q131B). Abdominal ultrasonography did not show any liver or spleen involvement.

Item Q131A: Left neck mass described for the boy in the vignette. Courtesy of A. Noor

Item Q131B: Ultrasonography findings for the boy in the vignette. Courtesy of A. Noor

Laboratory testing at diagnosis reveals the following: Laboratory Test Result White blood cell count 15,200/µL (15.2 × 10 9/L) Neutrophils 93% Lymphocytes 5% Monocytes 2% Hemoglobin 13.3 g/dL (133 g/L) Platelet count 269 × 103/µL (269 × 10 9/L) C-reactive protein 1.2 mg/dL (12 mg/L) Bartonella henselae IgG 1:64 Bartonella henselae IgM 0.2 mg/dl per hour thereafter. • Phototherapy before discharge • Parent or sibling requiring phototherapy or exchange transfusion • Family history or genetic ancestry suggestive of inherited red blood cell disorders, including glucose-6-phosphate dehydrogenase (G6PD) deficiency • Exclusive breastfeeding with suboptimal intake • Down syndrome • Macrosomic infant or a diabetic mother Reprinted with permission from Kemper AR, Newman TB, Slaughter JL, et al. Clinical practice guideline revision: management of hyperbilirubinemia in the newborn infant 35 or more weeks of gestation. Pediatrics 2022;e2022058859.

Early detection and treatment of hyperbilirubinemia are important to prevent bilirubin neurotoxicity, specifically to prevent the chronic neurologic sequelae known as bilirubin-induced neurologic dysfunction (BIND). A high level of bilirubin or an insufficient amount of serum protein to bind the bilirubin leads to circulating free bilirubin, which can easily penetrate the blood–brain barrier (BBB), especially when the BBB is disturbed because of the risk factors noted in Item C133C. This condition leads to hyperbilirubinemia neurotoxicity. Hyperbilirubinemia Neurotoxicity Risk Factors • Gestational age 5 mm prepubertal, >15 mm postpubertal) (Item C136B) • Freckling in the axillary or inguinal region • Cutaneous neurofibromas (≥2) • Lisch nodules (≥2) or choroidal abnormalities (≥2) • Plexiform neurofibroma (≥1) • Optic pathway glioma • Osseous lesion (sphenoid dysplasia, anterolateral tibial bowing, or long bone pseudoarthrosis) Pathogenic variant in the NF1 gene C. • • • • • • •

In the presence of a parent with diagnosed NF1, 1 of the previous features is sufficient to make a clinical diagnosis. Various clinical features seen in NF1 are age dependent: Café au lait macules and intertriginous freckling—commonly the first features seen in young children Optic pathway glioma—highest risk for development at 6 to 8 years of age Osseous lesions—congenital and can progress over time Cutaneous neurofibromas—more commonly seen during puberty in both males and females, with an increased emergence in pregnant females Plexiform neurofibromas of the face and neck—usually seen at birth Plexiform neurofibromas of other parts of the body—usually seen before adolescence

Pediatric surveillance includes annual dilated eye examination, complete physical examination, and developmental assessment. Although most tumors seen in individuals with NF1 are benign, there is a 10% lifetime risk of development of a malignant peripheral nerve sheath tumor from an underlying plexiform neurofibroma. Females with NF1 are at increased risk of developing breast cancer. The National Cancer Comprehensive Network recommends surveillance with annual American Academy of Pediatrics

463

PREP® Self-Assessment

PREPSA 2023

mammograms starting at 30 years of age. Learning disabilities and behavioral issues, such as attention-deficit/hyperactivity disorder, occur at a higher incidence in individuals with NF1.

Li-Fraumeni syndrome is an autosomal dominant tumor predisposition syndrome caused by a pathogenic variant in the tumor suppressor gene TP53. The condition predisposes the individual to a multitude of childhood and adultonset cancers, including but not limited to adrenocortical carcinoma, embryonal rhabdomyosarcoma, soft tissue sarcoma, acute lymphocytic leukemia, and brain tumors. Neurofibromatosis type 2 is an autosomal dominant tumor predisposition syndrome characterized by the development of vestibular schwannomas. Affected individuals are also at risk for the development of cranial and peripheral nerve schwannomas, meningiomas, ependymomas, and astrocytomas. Tuberous sclerosis complex involves multiple organ systems and can have the following manifestations: • Brain: Subependymal nodules, subependymal giant cell astrocytomas, seizures, cortical dysplasia, intellectual disability, developmental delay • Skin: Hypomelanotic macules, shagreen patch, facial angiofibromas, confetti-type hypopigmented skin lesions, fibrous cephalic plaque, ungual and gingival fibromas • Kidney: Cysts, angiomyolipomas, renal cell cancer • Heart: Rhabdomyomas • Lungs: Lymphangioleiomyomatosis American Academy of Pediatrics

464

PREP® Self-Assessment

PREPSA 2023

The personal and family history for the girl in the vignette is not consistent with these other tumor predisposition syndromes. PREP Pearls • Neurofibromatosis type 1 is an autosomal dominant tumor predisposition syndrome with complete penetrance. • Neurofibromatosis type 1 is characterized by development of café au lait macules, axillary or inguinal freckling, cutaneous neurofibromas, plexiform neurofibroma, optic pathway glioma, and osseous lesions. • Various clinical features seen in neurofibromatosis type 1 are age dependent. ABP Content Specifications(s) • Recognize the inheritance pattern of neurocutaneous hamartoses (eg, neurofibromatosis) • Recognize the clinical findings associated with neurofibromatosis in patients of various ages Suggested Readings • Adam MP, Ardinger HH, Pagon RA, et al, eds. Neurofibromatosis 1. GeneReviews [internet]. Seattle: University of Washington, Seattle; 1993-2022. • Al-Owain M, Faden M, Chedrawi A. Visual diagnosis: a child who has hyperpigmented spots and a forearm deformity. Pediatr Rev. 2009;30(5):182-186. doi:10.1542/pir.30.5.182. • Dove DE, Smith ML. Neurocutaneous syndromes. In: McInerny TK, Adam HM, Campbell DE, DeWitt TG, Foy JM, Kamat DM, eds. American Academy of Pediatrics Textbook of Pediatric Care. American Academy of Pediatrics; 2021:chap 297. Accessed September 1, 2022. Pediatric Care Online. • Korf BR, Bebin EM. Neurocutaneous disorders in children. Pediatr Rev. 2017;38(3):119128. doi:10.1542/pir.2015-0118. • Legius E, Messiaen L, Wolkenstein P, et al; International Consensus Group on Neurofibromatosis Diagnostic Criteria (I-NFDC). Revised diagnostic criteria for neurofibromatosis type 1 and Legius syndrome: an international consensus recommendation. Genet Med. 2021;23(8):1506-1513. doi:10.1038/s41436-021-01170-5 . • Miller DT, Freedenberg D, Schorry E, et al; Council on Genetics, American College of Medical Genetics and Genomics. Health supervision for children with neurofibromatosis type 1. Pediatrics. 2019;143(5):e20190660. doi:10.1542/peds.2019-0660.

American Academy of Pediatrics

465

PREP® Self-Assessment

PREPSA 2023

Question 137 An 8-month-old infant undergoing treatment for acute lymphoblastic leukemia is seen in the emergency department for evaluation of fever, diarrhea, and a generalized macular, erythematous, blanching, nonpruritic rash. Laboratory evaluation findings are significant for transaminitis. After further evaluation, the infant’s findings are determined to be secondary to a packed red blood cell transfusion administered 2 weeks ago. Of the following, the blood product preparation strategy MOST likely to have prevented this infant’s findings is A. irradiation B. leukoreduction C. selection of the freshest product D. washing

American Academy of Pediatrics

466

PREP® Self-Assessment

PREPSA 2023

Correct Answer: A Fever, transaminitis, diarrhea, and generalized macular erythematous, blanching, nonpruritic rash, as seen in the infant in the vignette, are common findings in many conditions, including viral infections and medication reactions. However, in the context of a recent blood product (packed red blood cells [PRBCs]) transfusion, transfusionassociated graft-vs-host disease (TAGVHD) is this infant’s most likely diagnosis. Acute graft-vs-host disease is a rare complication of a blood transfusion with a high mortality rate. The findings usually develop by 2 to 4 weeks after the transfusion. The initial symptom is often fever, which may develop in the first week. Rash and gastrointestinal symptoms and signs (eg, anorexia, nausea, vomiting, diarrhea, hepatomegaly, transaminitis, and direct hyperbilirubinemia) follow. Finally, pancytopenia may develop. The cause of death is usually an overwhelming infection. Because the signs and symptoms of TA-GVHD mimic many other conditions, delays in the diagnosis are common. Transfusion-associated graft-vs-host disease results when donor T cells proliferate in an immunocompromised host who is unable to reject them. Any blood product with mature, immunocompetent donor T lymphocytes can cause TAGVHD (PRBCs, platelets, granulocytes, and fresh, never-frozen plasma). Children at risk for TA-GVHD include those with an immunodeficiency (eg, severe combined immunodeficiency), newborns (because of their immature immune system), and children who have received immunosuppressive treatment (eg, chemotherapy). There is no adequate treatment for TA-GVHD. Medications that have been tried include corticosteroids, cyclosporine, anti-CD3 monoclonal antibody, and serine protease inhibitors that inhibit cytotoxic T cells. Treatment with a recently available anti–interleukin 2 receptor antibody (daclizumab) may be considered. Prevention of TA-GVHD is key because of the high mortality risk and lack of effective treatment. Irradiation of PRBCs eradicates the donor lymphocytes, making TA-GVHD less likely to occur. However, irradiation reduces the shelf-life of the PRBCs to approximately 1 month. If the PRBCs are stored after irradiation, there is an increase in the potassium content of the product. Therefore, it is best to irradiate the blood product immediately before use, if possible, or to use the product within 28 days of irradiation. Fresh PRBCs have a high amount of donor T cells. Although it is preferred that young children receive the freshest product, irradiation is needed to eliminate donor lymphocytes. Leukoreduction, which has been almost universally adapted for all blood products, significantly reduces the number of white blood cells in PRBCs. This practice decreases adverse transfusion reactions (such as fevers) but does not prevent TA-GVHD. Washing of RBCs eliminates products that accumulate during storage (eg, potassium and cytokines). It does not reduce the amount of donor T cells, which lead to TA-GVHD.

American Academy of Pediatrics

467

PREP® Self-Assessment

PREPSA 2023

PREP Pearls • Transfusion-associated graft-vs-host disease has a high mortality rate. • Irradiation of blood products administered to an immunodeficient individual is paramount in preventing transfusionassociated graft-vs-host disease. ABP Content Specifications(s) • Recognize the clinical features of graft-versus-host disease (transfusion related) Suggested Readings • Lanzkowsky P, Lipton J, Fish JD. Lanzkowsky’s Manual of Pediatric Hematology and Oncology. 6th ed. Elsevier Inc; 2016:729-732. • Ozgonenel B, Nash TA, Rajpurkar M. Blood components for pediatric transfusions. Pediatr Rev. 2020;41(5):259-261. doi:10.1542/pir.2018-0306. • Schroeder ML. Transfusion-associated graft-versus-host disease. Br J Haematol. 2002;117(2):275-287. doi:10.1046/j.1365-2141.2002.03450.x.

American Academy of Pediatrics

468

PREP® Self-Assessment

PREPSA 2023

Question 139 A 14-year-old adolescent boy is seen for concerns about a worsening chest deformity. He is selfconscious about his appearance and has experienced difficulty exercising. He has no other known health problems. On physical examination, his oxygen saturation in room air is 100% by pulse oximetry. There is a moderately severe depression of his sternum with some molding of the anterior ribs (Item Q139). Breath sounds are equal with no stridor or wheeze; there is good air exchange. Cardiac sounds are displaced slightly to the left. The remainder of the adolescent’s physical examination findings are normal.

A chest radiograph shows a long, narrow thoracic cavity with the heart slightly displaced to the left on anteriorposterior view and the sternum displaced posteriorly on the lateral view. Complete lung function testing including spirometry and lung volumes is normal for age and height. Of the following, the BEST test to evaluate the impact of this adolescent’s condition on his respiratory function is A. cardiopulmonary exercise testing B. computed tomography scan of the chest C. echocardiography D. a lung ventilation-perfusion scan

American Academy of Pediatrics

469

PREP® Self-Assessment

PREPSA 2023

Correct Answer: A The adolescent in the vignette has a pectus excavatum chest deformity that is causing him concern about his appearance and exercise capacity. The most common chest wall deformity, pectus excavatum often has a significant negative impact on body image and perceived exercise capacity. Static pulmonary function testing results are rarely abnormal, as is the case for the boy in the vignette. Cardiopulmonary exercise testing may uncover subtle deficits in exercise capacity and is the best test to further evaluate his respiratory function. Surgical repair of pectus excavatum has been shown to increase lung volumes, air flow, and exercise capacity to within the normal range on pulmonary function and cardiopulmonary exercise testing, but the greatest improvement is in body image perception and subjective assessment of exercise performance. The degree of pectus excavatum can be quantitated on computed tomography (CT) of the chest using the Haller index: the ratio of the internal transverse rib-to-rib diameter to the distance between the spine and sternum on a cross-sectional CT view (Item C139). An index of 2.5 or less is normal; the higher the index, the worse the pectus deformity. However, the Haller index is a poor biomarker of functional limitation; there is no direct correlation between severity as determined by Haller index and any pulmonary function abnormalities.

More severe pectus deformities can cause displacement of the heart to the left and cardiac conduction abnormalities. Exercise intolerance may have both cardiac and pulmonary components. While echocardiography is indicated in the evaluation of exercise intolerance in children and adolescents with pectus excavatum, this test will not define any functional respiratory deficits caused by the chest wall deformity. A lung ventilation-perfusion scan is not indicated for the adolescent in the vignette. Chest wall deformities do not cause ventilation-perfusion mismatch. American Academy of Pediatrics

470

PREP® Self-Assessment

PREPSA 2023

The second most common chest wall abnormality is scoliosis, which has many etiologies. Idiopathic scoliosis is rarely associated with objective pulmonary function changes until it is very severe. A restrictive pulmonary physiology is expected in any individual with scoliosis, with loss of lung volume on the side of the concavity and a hyperexpanded lung associated with stretched and poorly compliant intercostal muscles on the convex side. A rotational component and/or kyphosis worsens chest wall compliance and function, further restricting lung volume and chest wall expansion with respiration. When scoliosis is secondary to neuromuscular conditions, the underlying poor function of the intercostal muscles and diaphragm compounds the restrictive pulmonary physiology. There is a direct relationship between the severity and complexity of scoliosis and restrictive pulmonary physiology. Surgical management of severe or progressive scoliosis can stabilize lung function and prevent further decline; however, it is rarely associated with improvement in lung function even when the chest wall abnormalities are corrected. Early severe scoliosis is associated with structural abnormalities of the spine (eg, hemivertebrae, bar vertebrae) with or without rib fusions or absent ribs. Affected children may have lifethreatening pulmonary restriction from birth and require ventilator support until intervention can be provided to stabilize and expand the chest wall. This constellation of anatomic anomalies and severe restrictive pulmonary physiology (thoracic insufficiency syndrome) requires surgical intervention to preserve linear growth. If linear growth is not preserved, pulmonary restriction will worsen as the child ages. Placement of expanding rods in the spine for chest/rib stabilization promotes ongoing skeletal growth through interval extension. PREP Pearls • Severe chest wall deformities can cause restrictive pulmonary physiology. The severity of the restrictive lung disease is directly related to the complexity of the chest deformity. • Pectus excavatum can have a significant negative impact on self-image and perceived exercise capacity; static pulmonary function testing is usually normal. • Surgical repair of scoliosis is not likely to improve lung function but can prevent it from worsening. ABP Content Specifications(s) • Recognize the association of thoracic deformities with restrictive pulmonary disease

American Academy of Pediatrics

471

PREP® Self-Assessment

PREPSA 2023

Suggested Readings • Alapati D, Shaffer TH. Skeletal dysplasia: respiratory management during infancy. Respir Med. 2017;131:18-26. doi:10.1016/j.rmed.2017.07.063 . • Kelly RE Jr, Cash TF, Shamburger RC, et al. Surgical repair of pectus excavatum markedly improves body image and perceived ability for physical activity: multicenter study. Pediatrics. 2008;122(6):1218-1222. doi:10.1542/peds.20072723 . • Mayer OH, Allen JL. Chest wall and spinal deformities. In: Light MJ, ed. Pediatric Pulmonology. American Academy of Pediatrics; 2011:309-345. • Mino J, Stallion A, Monteiro R. Pectus excavatum and pectus carinatum. In: McInerny TK, Adam HM, Campbell DE, DeWitt TG, Foy JM, Kamat DM, eds. American Academy of Pediatrics Textbook of Pediatric Care. American Academy of Pediatrics; 2021:chap 309. Accessed September 1, 2022. Pediatric Care Online. • Redding GJ. Clinical issues for pediatric pulmonologists managing children with thoracic insufficiency syndrome. Front Pediatr. 2020;8:392. doi:10.3389/fped.2020.00392. • Yang S, Andras L, Redding G, Skaggs D. Early-onset scoliosis: a review of history, current treatment and future directions. Pediatrics. 2016;137(1). doi:10.1542/peds.20150709.

American Academy of Pediatrics

472

PREP® Self-Assessment

PREPSA 2023

Question 140 A 1-month-old infant is seen for a health supervision visit. His mother is concerned about his head shape. The infant was born at term after an unremarkable pregnancy and delivery. He has been well and is meeting age-appropriate developmental milestones. His family history is unremarkable. The infant’s growth parameters are normal, including head circumference. His physical examination findings are significant only for the head shape shown in Item Q140.

Item Q140: Head shape of the infant described in the vignette. Reprinted with permission from Dias MS, Samson T, Rizk EB, Governale LS, Richtsmeier JT and Section on Neurologic Surgery, Section on Plastic and Reconstructive Surgery. Identifying the misshapen head. Pediatrics. 2020;146(3):e2020015511.

Of the following, the BEST next management step for this infant is A. helmet therapy B. neurosurgery referral C. physical therapy D. reassurance

American Academy of Pediatrics

473

PREP® Self-Assessment

PREPSA 2023

Correct Answer: D The infant in the vignette has bathrocephaly resulting from a persistent mendosal suture (Item C140A). This suture usually disappears by the time of birth but when persistent can cause a prominent occiput. Bathrocephaly usually resolves without intervention, so the infant’s mother can be reassured that her son’s head shape will likely normalize. He does not require helmet therapy, referral to neurosurgery, or physical therapy for correction.

Persistent mendosal suture (arrowheads)

Bathrocephaly may be confused with sagittal synostosis, which is caused by abnormal fusion of the 2 parietal bones (Item C140B). Sagittal synostosis results in frontal bossing, a midline sagittal ridge, and narrowing of the biparietal diameter, features not associated with bathrocephaly. Infants with sagittal synostosis should be referred to neurosurgery as soon as possible (ideally by 6 to 12 weeks of age); late referral (6 to 10 months of age or later) is associated with the need for significantly more extensive surgical correction. Helmet therapy is usually also recommended after surgical correction of sagittal synostosis.

Item C140B: Sagittal synostosis

American Academy of Pediatrics

474

PREP® Self-Assessment

PREPSA 2023

Dolichocephaly can also be confused with sagittal synostosis. This condition occurs most frequently in premature infants in the neonatal intensive care unit. While these infants have an elongated skull, they do not have biparietal diameter narrowing. Early identification of a concerning head shape is crucial to ensure that infants receive treatment before the window of opportunity closes. Furthermore, identification of benign variations in skull shape can avoid unnecessary referrals and ease parental anxiety. One of the most common benign variations of head shape is occipital positional deformation (positional plagiocephaly), which most commonly results from supine positioning. Unilateral flattening of the occiput results in plagiocephalic changes with a parallelogram head shape (Item C140C). Bilateral flattening results in positional brachycephaly. Treatment involves repositioning education and physical therapy. For infants with moderate to severe positional deformation or older infants, helmet therapy can be beneficial.

Benign variations in head shape must be differentiated from certain types of craniosynostosis. Craniosynostosis is the premature closure of 1 or more cranial sutures and abnormal fusion of the contiguous bones that border the suture. Depending on which suture line is involved, head shape is altered, as shown in Item C140D.

American Academy of Pediatrics

475

PREP® Self-Assessment

PREPSA 2023

Unicoronal synostosis (anterior plagiocephaly) and bicoronal synostosis (anterior brachycephaly) are caused by fusion of the frontal and parietal bones and may be mistaken as occipital positional deformation. However, anterior plagiocephaly results in a trapezoidal head shape (Item C140E) and the radiographic changes shown in Item C140F. Anterior brachycephaly results in changes shown in Item C140G. Lambdoid synostosis may also be mistaken for occipital positional deformation. This condition is extremely rare and results in a head shape similar to a trapezoid instead of a parallelogram (Item C140H). Metopic synostosis results from prematurely fused frontal bones, which produces a prominent metopic ridge and trigonocephaly (Item C140I). American Academy of Pediatrics

476

PREP® Self-Assessment

PREPSA 2023

The least common craniosynostosis is frontosphenoidal craniosynostosis and produces the changes shown in Item C140J.

Anterior plagiocephaly

Unilateral lambdoid synostosis

Trigonocephaly

Management of most cases of craniosynostosis requires early referral to neurosurgery. Head computed tomography is performed to assist in surgical planning. Some types of craniosynostosis are syndromic; genetics consultation may be helpful to identify potential involvement of other organ systems. Children with developmental concerns require developmental/behavioral support. Craniosynostosis is a rare cause of microcephaly, defined as a head circumference less than the 5th percentile. Common causes of microcephaly are listed in Item C140K. Children with microcephaly, especially those with deceleration of growth, abnormal physical examination findings, and abnormal development should be referred for further evaluation. American Academy of Pediatrics

477

PREP® Self-Assessment

PREPSA 2023

Frontosphenoidal craniosynostosis Item C140K: Conditions Causing Microcephaly • Primary microcephaly o Chromosomal disorders o Anencephaly o Encephalocele o Holoprosencephaly o Agenesis of the corpus callosum o Neuronal migration disorders o Microcephaly vera • Secondary microcephaly o Intrauterine infections o Intrauterine toxins o Intrauterine vascular insufficiency o Hypoxic-ischemic brain injury o Intracranial hemorrhage o Neonatal infections (meningitis and o encephalitis) o Neonatal stroke o Chronic cardiopulmonary or renal disease

American Academy of Pediatrics

478

PREP® Self-Assessment o o

PREPSA 2023

Malnutrition Craniosynostosis

Adapted from Pina-Garza J. Fenichel’s Clinical PediatricmNeurology. 2nd ed. Amsterdam, Netherlands: Elsevier; 013:359.

PREP Pearls • Bathrocephaly, which is caused by a persistent mendosal suture and results in a prominent occiput, usually resolves without intervention. • Early identification of concerning head shape is crucial so that infants receive treatment before the window of opportunity closes. • Craniosynostosis is a rare cause of microcephaly. MOCA-Peds Objective • Evaluate an infant with microcephaly. ABP Content Specifications(s) • Plan the management of a patient with an abnormal head shape and/or growth (eg, craniosynostosis, plagiocephaly, microcephaly, macrocephaly) • Differentiate between normal and abnormal variations in head shape and/or growth (eg, craniosynostosis, plagiocephaly, microcephaly, macrocephaly) • Differentiate among the possible causes of abnormal head shape and/or growth (eg, craniosynostosis, plagiocephaly, microcephaly, macrocephaly) Suggested Readings • Dias MS, Samson T, Rizk EB, Governale LS, Richtsmeier JT and Section on Neurologic Surgery, Section on Plastic and Reconstructive Surgery. Identifying the misshapen head: craniosynostosis and related disorders. Pediatrics. 2020;146(3):e2020015511. doi:10.1542/peds.2020-015511. • Flannery AM, Tamber MS, Mazzola C, et al. Congress of neurological surgeons systematic review and evidence-based guidelines for the management of patients with positional plagiocephaly: executive summary. Neurosurgery. 2016;79(5):623-624. doi:10.1227/NEU.0000000000001426. • Kamat DM, Ahmer A. Positional deformational plagiocephaly. In: McInerny TK, Adam HM, Campbell DE, DeWitt TG, Foy JM, Kamat DM, eds. American Academy of Pediatrics Textbook of Pediatric Care. American Academy of Pediatrics; 2021:chap 316. Accessed September 1, 2022. Pediatric Care Online. • Purugganan OH. Microcephaly. In: McInerny TK, Adam HM, Campbell DE, DeWitt TG, Foy JM, Kamat DM, eds. American Academy of Pediatrics Textbook of Pediatric Care. American Academy of Pediatrics; 2021:chap 178. Accessed September 1, 2022. Pediatric Care Online.

American Academy of Pediatrics

479

PREP® Self-Assessment

PREPSA 2023

Question 141 A 15-year-old adolescent boy is brought to the office for a health supervision visit. His maternal grandfather received a kidney transplant secondary to autosomal dominant polycystic kidney disease at age 58 years. One year ago, the adolescent underwent renal ultrasonography screening that showed multiple bilateral cysts, the largest measuring 2 cm in the left kidney and 1.5 cm in the right kidney. The boy and his parents have no concerns today. He is doing well in school. He has not had headaches, dizziness, flank pain, swelling in any parts of his body, increased urinary frequency, or dysuria. The adolescent’s weight is at the 25th percentile, height is at the 40th percentile, and blood pressure is 120/72 mm Hg. The remainder of his physical examination findings are unremarkable. Of the following, the BEST next step in this adolescent’s evaluation is to perform A. computed tomography of the brain B. Doppler ultrasonography of the liver C. genetic testing D. urinalysis with microscopy

American Academy of Pediatrics

480

PREP® Self-Assessment

PREPSA 2023

Correct Answer: D The adolescent in the vignette most likely has autosomal dominant polycystic kidney disease (ADPKD). He has multiple cysts visualized on renal ultrasonography and a family history of ADPKD. While he remains asymptomatic, he should be screened with annual blood pressure measurements (to monitor for hypertension) and urinalysis with microscopy (to detect proteinuria and hematuria). Annual blood pressure measurement and urinalysis should be performed for all asymptomatic children with a family history of ADPKD. Serum creatinine values are usually normal in children with ADPKD and kidney function does not typically worsen until adulthood. Autosomal dominant polycystic kidney disease is the most common inherited kidney disease, with an incidence of 1 in 1,000 live births. The cysts gradually enlarge and replace the normal renal parenchyma leading to kidney failure. The signs and symptoms of ADPKD usually appear at age 30 to 40 years; these include hypertension, hematuria, proteinuria, flank pain, kidney stones, and urinary tract infection. Most children with ADPKD are asymptomatic. They are typically diagnosed incidentally on an imaging study or during screening due to family history. Autosomal dominant polycystic kidney disease can manifest in the neonatal period with an abdominal mass. Children may occasionally present before age 10 years with hypertension, proteinuria, or a renal concentrating defect (eg, polyuria, polydipsia); some may experience early progression to end-stage renal disease. Extrarenal manifestations, rarely seen in children, may include cysts in other organs (eg, liver, pancreas, and seminal vesicle), cerebral aneurysm, mitral valve prolapse, and colonic diverticula. Cerebral aneurysm is a rare manifestation that can present with a life-threatening intracranial bleed during adulthood. In children, the diagnosis of ADPKD is based on family history and the presence of one or more cysts on renal ultrasonography. Autosomal dominant polycystic kidney disease is associated with macroscopic cysts (>10 mm) on renal ultrasonography, while autosomal recessive polycystic kidney disease (ARPKD) is associated with numerous microscopic cysts. While computed tomography and magnetic resonance imaging are more sensitive for the detection of renal cysts, ultrasonography is the preferred imaging study because of the lack of radiation exposure and cost-effectiveness. Genetic testing is recommended for a child for whom the diagnosis of ADPKD is unclear (eg, equivocal imaging, inconsistent family history) or for a family member being considered for kidney donation. Because the boy in the vignette has a strong family history of ADPKD and characteristic renal ultrasonography findings, genetic testing is not essential and would not be the next step in management. A computed tomography scan of the brain to screen for cerebral aneurysm is not recommended for asymptomatic children since aneurysmal rupture is extremely rare. Doppler ultrasonography of the liver is helpful in diagnosing portal hypertension secondary to congenital hepatic fibrosis, which is associated with ARPKD. However, hepatic fibrosis is not associated with ADPKD and therefore, liver ultrasonography would not be indicated for the child in the vignette.

American Academy of Pediatrics

481

PREP® Self-Assessment

PREPSA 2023

Management of ADPKD includes treatment of hypertension and/or proteinuria with angiotensinconverting enzyme inhibitors or angiotensin receptor blockers, and treatment of urinary tract infections, kidney stones, and extra renal complications. Long-term monitoring of renal function, management of chronic kidney disease, and renal replacement therapy (dialysis or kidney transplant) may be required. PREP Pearls • Autosomal dominant polycystic kidney disease is the most common inherited kidney disease. • Most children with autosomal dominant polycystic kidney disease are asymptomatic and diagnosed incidentally on an imaging study or during screening due to family history. • Annual blood pressure measurement and urinalysis (to monitor for proteinuria and hematuria) should be performed for asymptomatic children with a family history of autosomal dominant polycystic kidney disease. ABP Content Specifications(s) • Recognize the clinical findings associated with autosomal-dominant polycystic kidney disease in patients of various ages • Plan the appropriate diagnostic evaluation for a patient in whom autosomal-dominant polycystic kidney disease is suspected Suggested Readings • Benun J, Lewis C. Polycystic kidney disease. Pediatr Rev. 2009;30(10):e78-e79; discussion e79. doi:10.1542/pir.30-10-e78 • De Rechter S, Breysem L and Mekahli D. Is autosomal dominant polycystic kidney disease becoming a pediatric disorder? Front Pediatr. 2017;5:272. doi:10.3389/fped.2017.00272. • Janjua HS, Lam KS, Gupta V, Krishna S. Congenital anomalies of the kidneys, collecting system, bladder, and urethra. Pediatr Rev. 2019;40(12):619-626. doi:10.1542/pir.20180242.

American Academy of Pediatrics

482

PREP® Self-Assessment

PREPSA 2023

Question 142 A 14-day-old, low-birth-weight neonate undergoing parenteral antibiotic therapy for bacterial sepsis in the neonatal intensive care unit becomes clinically unstable. Laboratory data are shown: Laboratory Test White blood cell count Hemoglobin Hematocrit Platelet count Sodium Potassium Chloride Carbon dioxide Blood urea nitrogen Creatinine Glucose

Result 18,000/µL (18.0 × 109/L) 8.7 g/dL (87 g/L) 25.8% 339 × 103/µL (339 × 10 9/L) 143 mEq/L (143 mmol/L) 4.8 mEq/L (4.8 mmol/L) 108 mEq/L (108 mmol/L) 23 mEq/L (23 mmol/L) 4.0 mg/dL (1.4 mmol/L) 10%) observed in most children younger than 4 years with acute EBV infection. The presence of fevers for more than 3 days should raise suspicion for multisystem inflammatory syndrome in children associated with COVID-19, which is seen 2 to 6 weeks after acute SARS-CoV-2 infection; however, this child has no known history suggestive of COVID-19 infection in that time frame. A diagnosis of incomplete Kawasaki disease should also be considered, but this child’s laboratory findings do not meet criteria for this diagnosis. Adenovirus, a common cause of viral infections in young children, is a non-enveloped DNA virus; 75% of children are seropositive by 5 years of age. It is transmitted through person-toperson contact (touching, kissing), droplets (cough, sneeze), and fomites (unclean hands, toys). The incubation period is 5 to 10 days. The clinical manifestations depend on the serotype, site of infection, and the child’s immune status. Several clinical syndromes are associated with adenoviral infection (Item C162B). Adenovirus infection in otherwise healthy children results in organ-specific (respiratory, gastrointestinal, genitourinary) self-limiting illness. Adenovirus conjunctivitis presents in 4 primary forms: (1) pharyngoconjunctival fever, (2) epidemic keratoconjunctivitis, (3) conjunctivitis associated with respiratory tract infection, or (4) isolated follicular conjunctivitis. Acute adenovirus gastroenteritis in young children typically lasts longer than other infectious causes of diarrhea. An exanthem (urticaria rash, maculopapular rash) may or may not accompany adenoviral clinical syndromes. Meningitis and encephalitis are occasionally seen. American Academy of Pediatrics

554

PREP® Self-Assessment

PREPSA 2023

Immunocompromised children, particularly those with a history of hematopoietic stem cell or solid organ transplant, are predisposed to severe, disseminated infections in the first month after the transplant. The preferred method for diagnosis of adenovirus infection is a molecular assay (eg, polymerase chain reaction) test performed on a nasopharyngeal swab sample. The commonly available respiratory multiplex viral panels include adenovirus. Treatment of immunocompetent hosts is primarily supportive care.

American Academy of Pediatrics

555

PREP® Self-Assessment

PREPSA 2023

PREP Pearls • Adenovirus is responsible for a wide variety of infections in children, most commonly respiratory and gastrointestinal. • Adenovirus pharyngoconjunctival fever presents with follicular conjunctivitis, tender preauricular lymphadenopathy, pharyngitis, and fever. • Adenovirus is diagnosed by polymerase chain reaction performed on a nasopharyngeal swab sample; this test is included in the available respiratory multiplex viral panels. ABP Content Specifications(s) • Understand the epidemiology of adenovirus • Recognize the clinical features associated with adenovirus infection Suggested Readings • Allen UD, Demmler GJ. Adenoviruses. In: Long SS, ed. Principles and Practice of Pediatric Infectious Diseases. 4th ed. Elsevier; 2012:1067-1071. • American Academy of Pediatrics. Adenovirus infections. In: Kimberlin DW, Barnett ED, Lynfield R, Sawyer MH, eds. Red Book: 2021–2024 Report of the Committee on Infectious Diseases. 32nd ed. American Academy of Pediatrics; 2021. Accessed September 1, 2022. Red Book Online. • Chen TK, Cherry JD. Adenoviruses. In: Cherry JD, Harrison GJ, Kaplan SL, Hotez PJ, Steinbach WJ, eds. Feigin and Cherry’s Textbook of Pediatric Infectious Diseases. 7th ed. Saunders; 2014:1364-1383. • Steele RW. Pharyngitis and tonsillitis. In: McInerny TK, Adam HM, Campbell DE, DeWitt TG, Foy JM, Kamat DM, eds. American Academy of Pediatrics Textbook of Pediatric Care. American Academy of Pediatrics; 2021:chap 311. Accessed September 1, 2022. Pediatric Care Online.

American Academy of Pediatrics

556

PREP® Self-Assessment

PREPSA 2023

Question 163 A 7-year-old girl is brought to the clinic for concerns about her balance. For the past few days, the girl has complained of the room spinning and nausea. She has 5 to 6 episodes per day, each lasting about 1 minute, during which she grabs onto a chair or the sofa. Between episodes she acts and feels normal. She is otherwise healthy with no history of trauma or recent viral symptoms. Her physical examination findings are unremarkable, including her tympanic membranes and a detailed neurologic examination. Of the following, the BEST next step in this girl’s management is A. an antihistamine prescription B. head and neck imaging C. observation D. referral to neurology

American Academy of Pediatrics

557

PREP® Self-Assessment

PREPSA 2023

Correct Answer: C The girl in the vignette most likely has benign paroxysmal vertigo of childhood (BPVC), given her intermittent symptoms and unremarkable physical examination findings. Benign paroxysmal vertigo of childhood is typically a self-limited condition for which observation is recommended. Diagnostic evaluation and referral to neurology are only recommended if symptoms worsen or there is impairment of function. Medication management with antiemetics or antihistamines may be warranted if episodes are long lasting, which is not yet the case for the girl in the vignette. Although the cause of BPVC is unknown, there is usually a family history of migraine headaches, and children with BPVC may subsequently develop migraines. Young children may have difficulty describing vertigo, defined as a sensation of rotational movement that usually involves the vestibular system, either peripheral (inner ear) or central (brainstem, cerebellum, or cranial nerve VIII). History and physical examination findings provide etiologic clues. Middle ear disease is the most common cause of vertigo in children. Benign paroxysmal vertigo of childhood is the second most common cause of vertigo in children aged 2 to 12 years. The diagnosis of BPVC is clinical and based the following criteria: • Brief episodes of vertigo associated with vomiting, pallor, nystagmus, ataxia, or fearfulness • Normal neurologic examination, audiometric, and vestibular findings between episodes • Symptom-free intervals • Not attributable to another disorder Benign paroxysmal vertigo of childhood may be considered a migraine equivalent. The prognosis is good, usually with spontaneous resolution before adolescence. Benign paroxysmal vertigo of childhood is a distinct entity from benign paroxysmal positional vertigo (BPPV), which is associated with position change, can be elicited by the Dix-Hallpike maneuver (neck is extended and turned to one side followed by quick placement in the supine position), and is caused by calcium deposits in the posterior semicircular canal. Benign paroxysmal positional vertigo is rare in children. Vestibular migraines may also cause vertigo. PREP Pearls • Benign paroxysmal vertigo of childhood is the second most common cause of vertigo in children aged 2 to 12 years; the most common cause is middle ear disease. • The diagnosis of benign paroxysmal vertigo is based on a history of brief episodes of vertigo associated with vomiting, pallor, nystagmus, ataxia or fearfulness, normal neurologic examination findings, audiometric, and vestibular findings between episodes, symptom-free intervals, and no other attributable disorder. • Benign paroxysmal vertigo is a self-limited condition for which the treatment is observation. Referral to neurology for further evaluation and management may be considered for worsening symptoms or impairment of function. ABP Content Specifications(s) • Recognize the clinical findings associated with benign paroxysmal vertigo American Academy of Pediatrics

558

PREP® Self-Assessment

PREPSA 2023

Suggested Readings • Batson G. Benign paroxysmal vertigo of childhood: a review of the literature. Paediatr Child Health. 2004;9(1):31-34. doi:10.1093/pch/9.1.31. • Benun J. Balance and vertigo in children. Pediatr Rev. 2011;32(2):84-85. doi:10.1542/pir.32.2.84. • Rivera RF, Sellinger CR. Dizziness and vertigo. In: McInerny TK, Adam HM, Campbell DE, DeWitt TG, Foy JM, Kamat DM, eds. American Academy of Pediatrics Textbook of Pediatric Care. American Academy of Pediatrics; 2021:chap 140. Accessed September 1, 2022. Pediatric Care Online. • van de Berg R, Widdershoven J, Bisdorff A, et al. Vestibular migraine of childhood and recurrent vertigo of childhood: diagnostic criteria consensus document of the Committee for the Classification of Vestibular Disorders of the Bárány Society and the International Headache Society. J Vestib Res. 2021;31(1):1-9. doi:10.3233/VES-200003.

American Academy of Pediatrics

559

PREP® Self-Assessment

PREPSA 2023

Question 164 A 16-month-old girl is seen for a health supervision visit. She eats a wide variety of foods and drinks 40 oz of cow milk per day. On physical examination, she looks pale. Her vital signs are normal for age. The remainder of her physical examination findings are normal. Screening laboratory evaluation reveals a hemoglobin level of 6.4 g/dL (64 g/L) and a mean corpuscular volume of 65 fL. After review of the laboratory results, a more detailed history is obtained. The family recently renovated their house. The girl often puts items into her mouth (things found on the floor and paper items). She has not had bright red blood in her diaper or darker stools. Of the following, the MOST likely additional laboratory findings for this girl include

A B C D

Reticulocytes High Low Low Low

RDW Normal Normal High Normal

Ferritin Normal High Low Normal

American Academy of Pediatrics

Soluble Transfer Receptor Normal Low High High

560

PREP® Self-Assessment

PREPSA 2023

Correct Answers: C The girl in the vignette has iron-deficiency anemia, the most common cause of anemia in her age group. Her high-volume consumption of cow milk, pica, and pallor support this diagnosis. Laboratory findings consistent with irondeficiency anemia include a low hemoglobin (Hb) level, low mean corpuscular volume (MCV), low reticulocyte count, high red blood cell distribution width (RDW), high soluble transferrin receptor (sTfR) level, and low ferritin level (the storage form of iron). Although a low reticulocyte count is an inappropriate response to anemia (the appropriate response is increased red blood cell production), the reticulocyte count is decreased in irondeficiency anemia because of the lack of available iron to make new red blood cells. It is the first value to increase once iron is supplemented. The sTfR level helps differentiate iron-deficiency anemia (high sTIR) from anemia of chronic disease or inflammation (low sTIR). Anemia of chronic disease or inflammation is associated with a low reticulocyte count, normal RDW, high ferritin level, and low sTfR level. Gastrointestinal blood loss should be considered in the differential diagnosis of any child with iron-deficiency anemia. There may be a poor response to oral iron supplementation if there is a gastrointestinal condition; evaluation by a gastroenterologist for an inflammatory bowel disorder may be indicated. Markers of inflammation that can be helpful in determining the cause of anemia include elevated ferritin level, erythrocyte sedimentation rate, C-reactive protein level, and platelet count. Both α- and β-thalassemia traits are associated with mild microcytic anemia. In these conditions the reticulocyte count is low, RDW is normal, ferritin level is normal, and the sTfR is high. Iron study results are normal, and there is no improvement in the Hb level with iron supplementation. An elevation in HbA2 on Hb electrophoresis is consistent with β-thalassemia trait; however, the results can be falsely normal in the setting of iron deficiency. Hemoglobin electrophoresis cannot be used to diagnose α-thalassemia trait; it can sometimes be detected on newborn screening as Hb Barts. Genetic testing is available for both α- and β-thalassemia traits. Lead toxicity, an uncommon cause of anemia, is associated with a high reticulocyte count, normal RDW, normal ferritin level, and normal sTfR level. The MCV and reticulocyte count may be normal in lead poisoning. The peripheral blood smear may show basophilic stippling. Item C164A summarizes the laboratory findings associated with causes of microcytic anemia. The reticulocyte count can help determine the cause of anemia, especially normocytic anemia. An elevated reticulocyte count usually indicates destruction of red blood cells, either intravascular or extravascular. Additional laboratory findings that support hemolysis include an elevated indirect bilirubin level and low haptoglobin level. A positive direct antiglobulin (Coombs) test result supports an immune hemolytic anemia. A low reticulocyte count can indicate impaired bone marrow function, either from temporary suppression (eg, infection or medication) or bone marrow failure. American Academy of Pediatrics

561

PREP® Self-Assessment

PREPSA 2023

Normal Hb and MCV values vary with age. It is important to use the correct reference range to determine which laboratory findings warrant further investigation. Item C164B shows the normal range for Hb values and Item C164C shows the normal range for MCV values based on age.

American Academy of Pediatrics

562

PREP® Self-Assessment

American Academy of Pediatrics

PREPSA 2023

563

PREP® Self-Assessment

PREPSA 2023

PREP Pearls • Iron deficiency is the most common cause of microcytic anemia in childhood. • Children with iron deficiency should be evaluated to identify any gastrointestinal cause of blood loss (eg, inflammatory bowel disease). ABP Content Specifications(s) • Recognize the laboratory findings associated with microcytic anemia • Recognize the normal variations in hemoglobin concentration and mean cell volume during childhood • Distinguish between a disorder of erythrocyte production and a disorder of erythrocyte destruction based on laboratory results Suggested Readings • Krishnamurti L. Iron-deficiency anemia. In: McInerny TK, Adam HM, Campbell DE, DeWitt TG, Foy JM, Kamat DM, eds. American Academy of Pediatrics Textbook of Pediatric Care. American Academy of Pediatrics; 2021:chap 279. Accessed September 1, 2022. Pediatric Care Online. • Lanzkowsky P, Lipton J, Fish JD. Lanzkowsky’s Manual of Pediatric Hematology and Oncology. 6th ed. Elsevier Inc; 2016:38,77. • McFarren AK, Levy AS. Anemia and pallor. In: McInerny TK, Adam HM, Campbell DE, DeWitt TG, Foy JM, Kamat DM, eds. American Academy of Pediatrics Textbook of Pediatric Care. American Academy of Pediatrics; 2021:chap 128. Accessed September 1, 2022. Pediatric Care Online. • Richardson M. Microcytic anemia. Pediatr Rev. 2007;28(1):5-14. doi:10.1542/pir.28-1-5. • Zanetti R, Feldman B, Porea T. Microcytic anemia. Pediatr Rev. 2021;42(1):41-43. doi:10.1542/pir.2019-0295.

American Academy of Pediatrics

564

PREP® Self-Assessment

PREPSA 2023

Question 165 A neonate born at 31 weeks’ gestation with a birth weight of 1,500 g is admitted to the neonatal intensive care unit. The neonate currently requires continuous positive pressure support of 5 cm H2O and a fraction of inspired oxygen of 25%. To provide ease of access for care, the newborn is placed under a radiant warmer. Of the following, the MOST likely effect of this care environment on the neonate will be A. decreased insensible water loss B. excess weight gain C. hyperthermia D. hypoglycemia

American Academy of Pediatrics

565

PREP® Self-Assessment

PREPSA 2023

Correct Answer: C The most likely effect of care for a neonate under a radiant warmer is hyperthermia. Radiant warmers, while allowing for easy access to neonates, are associated with increased insensible water losses and, if not used properly, can result in significant deleterious effects. All newborns, especially those born prematurely and/or at low birth weight, are at higher risk for heat loss than older children because of neonates’ higher skin surface–area-to-weight (volume) ratio, decreased amount of subcutaneous fat, permeable skin, and increased transdermal loss of water and heat. These factors determine the neutral thermal environment (NTE), a set of conditions in which oxygen consumption (metabolic demand) is minimal when body temperature is maintained in the reference range. An increase or decrease in the NTE can have deleterious effects on the neonate. The NTE temperature range is dependent on the gestational age, weight, postnatal age, and clinical status of the neonate. Heat loss occurs via a combination of 4 different phenomena: evaporation, conduction, radiation, and convection (Item Q165A).

Reprinted with permission from Roychoudhury S, Yusuf K. Thermoregulation: Advances in Preterm Infants. Neoreviews. 2017;18(12):e695.

The NTE under a radiant warmer is managed through maintaining normal skin temperature by use of servo control with a skin sensor probe. If the sensor probe is not properly placed or is dislodged, it can result in hyperthermia or hypothermia. The probe should be taped to an exposed, fleshy part of the neonate’s trunk. A probe taped over a bony prominence or a limb can record a lower temperature than the actual general skin temperature; in response, the radiant warmer will provide additional heat, resulting in hyperthermia. If the neonate is lying on top of the probe, the sensor can register a temperature higher than the neonate’s actual temperature, causing the radiant warmer to provide less heat than needed, resulting in hypothermia. The automatic rather than manual mode is recommended to decrease the risk of hyperthermia. A American Academy of Pediatrics

566

PREP® Self-Assessment

PREPSA 2023

neonate’s insensible water loss (IWL) while under a radiant warmer can lead to weight loss; excess weight gain and hypoglycemia are not effects of this environment. Humidity plays an important role in the maintenance of NTE, especially in preterm neonates because of their immature epithelial layers, larger surface area, and increased skin vascularity. Inadequate humidity may contribute to morbidity and mortality. Thermal control is improved and IWL is decreased when adequate humidity is present in the environment. The daily fluid requirement for neonates, especially preterm neonates, depends largely on IWL (Item Q165B). The major component of IWL is transepidermal loss, which varies with gestational age, postnatal age, and ambient water vapor pressure. Insensible water loss also occurs across the upper airway epithelium. Insensible water loss is obligate free water loss, which can make electrolyte management a challenge. Antenatal corticosteroids decrease IWL by decreasing transepidermal loss. Item C165B. Estimated Newborn Fluid Requirement by Birth Weight.

Neonates cared for in double-walled incubators have less IWL compared with those cared for with radiant warmers, single-walled incubators, or heat shields. The level of humidity under a radiant warmer is dependent on the humidity of the room and cannot be controlled easily. Radiant warmers can increase IWL by 50% to 80%, whereas doublewalled incubators can decrease IWL by 30% to 50%. Double-walled incubators have dual modes: when closed, they allow care in a humidified environment, and while open, they provide access to the neonate for procedures, handling, and parental interactions with less control of humidification. A doublewalled incubator is the best choice of care environment for attaining a NTE for the neonate in the vignette. PREP Pearls • A neutral thermal environment is important for care of newborns. • Premature neonates should ideally be cared for in a double-walled incubator. • If a radiant warmer is used, care must be taken to ensure that the sensor probe is properly placed and the warmer is placed in automatic mode; careful monitoring of the neonate’s temperature is required. ABP Content Specifications(s) • Recognize the hazards and benefits associated with the use of radiant warmers for neonates American Academy of Pediatrics

567

PREP® Self-Assessment • •

PREPSA 2023

Understand how prematurity and the use of radiant warmers affect insensible water loss, especially in preterm infants Recognize the differences in daily fluid requirements per kilogram of body weight in preterm and full-term infants

Suggested Readings • American Academy of Pediatrics. Thermal environment. In: PCEP Book 1: Maternal and Fetal Evaluation and Immediate Newborn Care. American Academy of Pediatrics; October 2021:247-284. doi:10.1542/9781610024952-unit7. • Pinheiro JMB. Assessment and stabilization at delivery. In: McInerny TK, Adam HM, Campbell DE, DeWitt TG, Foy JM, Kamat DM, eds. American Academy of Pediatrics Textbook of Pediatric Care. American Academy of Pediatrics; 2021:chap 109. Accessed September 1, 2022. Pediatric Care Online. • Ringer SA. Core concepts: thermoregulation in the newborn part I: basic mechanisms. Neoreviews. 2013;14(4):e161– e167. doi10.1542/neo.14-4-e161. • Ringer SA. Core concepts: thermoregulation in the newborn, part II: prevention of aberrant body temperature. Neoreviews. 2013;14(5):e221–e226. doi:10.1542/neo.14-5e221.

American Academy of Pediatrics

568

PREP® Self-Assessment

PREPSA 2023

Question 166 A 3-year-old boy is seen in the office to establish care after emigrating from Southeast Asia 2 months ago. He has had a cough for 12 months and has been eating less for the past 6 months. He has a habit of soil pica. The family has 4 dogs and 5 cats. The boy’s physical examination findings are remarkable for scattered wheezing in both lung fields and a liver edge palpable 2 cm below the right costal margin. His weight is less than the third percentile. His parents report that his weight is usually at the 25th percentile. Laboratory data are shown: Laboratory Test White blood cell count Neutrophils Lymphocytes Monocytes Eosinophils Hemoglobin Platelet count Alanine aminotransferase Aspartate aminotransferase

Result 15,000/µL (15.0 × 10 9/L) 45% 30% 5% 20% 10 g/dL (100 g/L) 200 × 103/µL (200 × 10 9/L) 55 U/L 65 U/L

Of the following, the BEST next step to confirm this boy’s diagnosis is to obtain A. serum for Ascaris IgG B. serum for Toxocara IgG C. stool for bacterial culture D. stool for ova and parasites

American Academy of Pediatrics

569

PREP® Self-Assessment

PREPSA 2023

Correct Answer: B The boy in the vignette has toxocariasis, a tissue roundworm infection. His signs and symptoms (wheezing, hepatomegaly, anorexia, and weight loss), exposure to dogs and cats, habit of soil pica, and eosinophilia are consistent with infection with the visceral larva migrans form of Toxocara. In Southeast Asia, the seroprevalence of toxocariasis is approximately 34%. In the United States, the seroprevalence is 5%. The diagnosis of toxocariasis is based on compatible signs and symptoms and risk factors for acquisition, such as exposure to dogs and cats, which are definitive hosts. Young children who spend time in potentially contaminated areas (eg, playgrounds and sandboxes) and place soil in their mouths are at increased risk of acquiring Toxocara infection. Toxocara infection is confirmed with serologic testing, with an enzyme-linked immunosorbent assay that detects Toxocara IgG levels. Stool for ova and parasites is not helpful in the diagnosis of toxocariasis because the parasite does not complete its life cycle in the human gastrointestinal tract. Toxocariasis, a roundworm (nematode) infection, is caused by 2 species: Toxocara canis and Toxocara cati, parasites of dogs (puppies) and cats (kittens), respectively. Humans are unintentional hosts. Toxocara infection causes disease when larvae migrate to tissues and local immune reactions result in injury. Item C166A reviews the major symptomatic presentations.

American Academy of Pediatrics

570

PREP® Self-Assessment

PREPSA 2023

Albendazole is the recommended treatment for Toxocara. Mebendazole is the alternative. Testing the serum for Ascaris IgG would not be appropriate for the boy in the vignette, who has a clinical picture consistent with toxocariasis. Ascariasis (Ascaris lumbricoides) is the most common intestinal roundworm infection. Its clinical features overlap with toxocariasis. However, exposure to dogs is not a risk factor for Ascaris infection. A child typically acquires ascariasis through ingestion of water or food contaminated with the parasite’s eggs. The diagnosis is made by visualization of Ascaris ova (Item C166B) on stool microscopy. Serologic testing is reserved for seroprevalence studies rather than clinical diagnosis. Cross-reactivity of Ascaris IgG with other helminths is common. Item C166B: Ascaris ova

The boy in the vignette does not have signs or symptoms of gastroenteritis from a bacterial pathogen, such as Salmonella or Shigella. Therefore, obtaining a stool culture would not confirm his diagnosis. PREP Pearls • Toxocariasis is a parasitic infection caused by 2 species of roundworm (nematode): Toxocara canis and Toxocara cati, parasites of dogs (puppies) and cats (kittens), respectively. • Toxocara infection causes disease when larvae migrate to tissues and local immune reactions result in injury. Major symptomatic presentations are visceral larva migrans (young children) and ocular larva migrans (older children and adolescents). • The diagnosis of Toxocara is confirmed by Toxocara IgG serologic testing. Stool for ova and parasites are not helpful because the adult worm does not complete its life cycle in the human gastrointestinal tract. ABP Content Specifications(s) • Understand the epidemiology of Toxocara • Recognize the clinical features associated with Toxocara infestation

American Academy of Pediatrics

571

PREP® Self-Assessment

PREPSA 2023

Suggested Readings • Abdel-Haq N, Chearskul P, Rafee Y, Asmar BI. Parasitic infections. In: McInerny TK, Adam HM, Campbell DE, DeWitt TG, Foy JM, Kamat DM, eds. American Academy of Pediatrics Textbook of Pediatric Care. American Academy of Pediatrics; 2021:chap 308. Accessed September 1, 2022. Pediatric Care Online. • American Academy of Pediatrics. Toxocariasis. In: Kimberlin DW, Barnett ED, Lynfield R, Sawyer MH, eds. Red Book: 2021–2024 Report of the Committee on Infectious Diseases. 32nd ed. American Academy of Pediatrics; 2021. Accessed September 1, 2022. Red Book Online. • Cherry JD, Harrison GJ, Kaplan SL, Hotez PJ, Steinbach WJ. Parasitology. In: Feigin and Cherry’s Textbook of Pediatric Infectious Diseases. 7th ed. Sauders; 2014:section XXII. • Dickson D, Hotez PJ. Tissue nematodes. In: Long SS, ed. Principles and Practice of Pediatric Infectious Diseases. 4th ed. Elsevier; 2012:1334-1341.e2. • Woodhall DM. Fiore AE, Toxocariasis: a review for pediatricians. J Pediatric Infect Dis Soc. 2014;3(2):154-149. doi:10.1093/jpids/pit066.

American Academy of Pediatrics

572

PREP® Self-Assessment

PREPSA 2023

Question 167 A pediatric resident seeks to study the impact of mother–infant separation at birth on breastfeeding outcomes during the COVID-19 pandemic. The study population includes mothers with SARS-CoV-2 infection (symptomatic and asymptomatic) and their asymptomatic infants in 2 different community hospitals. All mother-infant dyads were separated at one hospital. In the other hospital, asymptomatic mother-infant dyads were allowed to room-in. The pediatric resident collects information on breastfeeding outcomes at 1 and 6 months after discharge in these 2 groups. Of the following, the design of this study is BEST described as A. case-control B. cohort C. cross-sectional D. ecologic

American Academy of Pediatrics

573

PREP® Self-Assessment

PREPSA 2023

Correct Answer: B Among the response choices, the study design that best describes the research study in vignette is a cohort study. Clinical research is of paramount importance in the development of evidence-based medical practice. Clinical research studies are divided into the categories of experimental and observational. The majority of new treatments and technologies are introduced into clinical practice after randomized prospective clinical trials (experimental research). On the other hand, observational research examines the effects of predetermined policies, treatments, and interventions. Item C167 provides an overview of the different types of observational studies.

In the vignette, the investigator seeks to study the impact of a predetermined policy of motherinfant separation on breastfeeding outcomes. The 2 hospitals have different policies. One separates mother–infant dyads if the mother is detected to have SARS-CoV-2, irrespective of symptom status. The other hospital allows rooming in of asymptomatic mother–infant dyads. The outcome of breastfeeding is observed overtime in the 2 groups. This is a longitudinal cohort study design, prospective in nature. A cohort study begins with the exposure (or risk factor); the cohort is observed over time for the development of the outcome of interest. In contrast, a case-control study begins with the outcome and looks back in time for the exposure/risk factor. A case-control study design is useful when the disease outcome is rare because it begins with the selection of a population with the outcome of interest. The case population has evidence of the outcome of interest, whereas the control population does not. An example of a case-control study is a study of children with asthma (cases) compared with children without asthma (controls); the study looks at whether the parents have a history of asthma (risk factor). A cross-sectional study collects data on both the exposure and outcome at a single point in time; study participants are not followed longitudinally. The prevalence of exposure in patients with and without the outcome can be generated with cross-sectional studies; however, causation cannot be ascertained. An example of a cross-sectional study is assessment of the prevalence of long-term COVID-19 symptoms in children at a given time. In an ecologic study, the outcome assessment is made at a population level rather than an individual level. For example, a study may compare the outcome rates between populations in 2 different geographic locations with different exposures. American Academy of Pediatrics

574

PREP® Self-Assessment

PREPSA 2023

In the hierarchy of levels of evidence, systematic reviews carry the highest level. A systematic review is a comprehensive summary of all available evidence on a specific subject. It requires a rigorous process, which involves the inclusion of studies based on defined eligibility criteria, incorporates critical appraisal, and uses statistical testing through meta-analysis. Meta-analysis combines the results of several studies. This method is used to synthesize the best available evidence using a clear and transparent approach. It considers the strengths as well as weaknesses of each study, taking into account populations and interventions, to assess specific outcomes. PREP Pearls • Clinical research is either experimental or observational. Experimental research includes randomized prospective trials (eg, drug or vaccine clinical trials). Observational research includes cross-sectional, case-control, and cohort studies. • In a cohort study, the measurement of exposure and outcome is made over time (longitudinal), whereas in a crosssectional study it is made at a specific point of time. • A cohort study begins with exposure to a risk factor and observation for the outcome of interest is made over time. A case-control study begins with cases (with outcome) and controls (without outcomes) who are then assessed for the presence of the defined risk factors/exposure. ABP Content Specifications(s) • Understand the uses and limitations of cross-sectional and longitudinal studies • Understand the uses and limitations of systematic review and meta-analysis Suggested Readings • Hammill BG. Observational study designs. In: Lopes RD, Harrington RA. eds. Understanding Clinical Research. McGraw Hill; 2013: chap 12. • Hulley SB. Cummings SR, Browner WS, Grady DG. Newman TB. In: Designing Clinical Research, 4th ed. Lippincott Williams & Wilkins; 2013: chap 7. • Johnson SL. A question of time: Cross-sectional versus longitudinal study designs. Pediatr Rev. 2010;31(6):250-251. doi:10.1542/pir.31.6.250. • LeBrun DG, Kocher MS, Baldwin KD, Patel NM. How often are study design and level of evidence misreported in the pediatric orthopaedic literature? J Pediatr Orthop. 2020; 40(5):e385-e389. doi: 10.1097/BPO.0000000000001470. • Mason A. Clinical drug trials. Pediatr Rev. 2018;39(3):150-152. doi:10.1542/pir.20160230.

American Academy of Pediatrics

575

PREP® Self-Assessment

PREPSA 2023

Question 168 A 2-hour-old neonate born at 36 4/7 weeks’ gestation is examined in the newborn nursery. She was born to a 28year-old gravida 2, para 1 woman in a normal spontaneous vaginal delivery. Maternal prenatal testing is negative for group B Streptococcus, and rupture of membranes occurred 20 hours before delivery. The mother had a temperature of 39.6°C measured 8 hours before delivery and fetal tachycardia was noted at that time. The neonate has a temperature of 37.1°C, heart rate of 130 beats/min, and respiratory rate of 48 breaths/min. She appears vigorous with a good cry and normal tone. The remainder of her physical examination findings are normal. The mother requests early discharge. Of the following, the BEST next management step for this neonate is to A. consider early discharge if the neonate's vital signs are stable with next day follow-up B. observe closely for signs of sepsis for at least 48 hours C. obtain a complete blood cell count and C-reactive protein level, and perform blood culture D. obtain a complete blood cell count, perform blood culture, and start empiric antibiotics

American Academy of Pediatrics

576

PREP® Self-Assessment

PREPSA 2023

Correct Answer: D The neonate in the vignette is at risk for sepsis due to a combination of factors, late preterm gestation, maternal fever, and prolonged rupture of membranes (PROM, >18 hours). The best next step in management is to obtain a complete blood cell count, perform a blood culture, and start intravenous ampicillin and gentamicin treatment. Early discharge is not the best option because signs of sepsis may be delayed until 72 hours after birth. Close observation for 48 hours without empiric antibiotics would not be a good choice given this neonate’s multiple risk factors for sepsis. Although normal laboratory evaluation may seem reassuring, the white blood cell counts and inflammatory markers may not be a reliable indicator of sepsis in the first 24 to 48 hours after birth. Although the incidence of early-onset sepsis (EOS) has decreased over the past several years with the use of intrapartum antibiotics, the morbidity and mortality of affected neonates remains high. Clinicians must weigh the risk of delayed treatment of neonatal sepsis with that of unnecessary antibiotic use. Risk factors for EOS include prematurity, PROM, maternal colonization with group B Streptococcus (GBS), and evidence of maternal chorioamnionitis. A suspected intra-amniotic infection is defined by the American College of Obstetricians and Gynecologists as maternal fever (>39°C) and at least 1 of the following: maternal leukocytosis, purulent cervical discharge, and fetal tachycardia. If the mother meets the criteria for suspected intra-amniotic infection, laboratory evaluation and empiric treatment with antibiotics are recommended for neonates with any of the following: • Signs of sepsis (eg, ill appearing) • Gestational age 1 minute, and concerning history and physical examination findings, he does not meet criteria for lower-risk BRUE and further evaluation is indicated. Reassurance and follow-up is not appropriate. Potential life-threatening etiologies for his presentation include central nervous system (CNS) abnormalities and abusive head trauma. Thus, the best next step in his management is to obtain computed tomography of the head. A high index of suspicion for nonaccidental trauma should be maintained. Concerning findings include the infant’s fussiness and the bruise on his arm in the context of an otherwise unexplained recurrent potentially life-threatening event. Nothing in the infant’s history or physical examination suggests a viral respiratory infection, so a nasal swab for respiratory syncytial virus is not indicated. Home apnea monitoring is not appropriate at this time. Further evaluation would be required to identify an underlying abnormality for which home monitoring is appropriate. Up to 43% of all infants may have a BRUE in the first year after birth, defined as having one or more of the following components: • Cyanosis or pallor • Absent, decreased, or irregular breathing • Marked change in alertness or level of responsiveness A thorough history, social history, and physical examination are required for infants with a suspected diagnosis of BRUE. For those meeting the American Academy of Pediatrics criteria for lower-risk BRUE, reassurance and follow-up are appropriate. Criteria for lower-risk BRUE include (all criteria must be met): • Age >60 days • Gestational age ≥32 weeks • Single episode without recurrence (first ever and not occurring in clusters) • Duration 3 seconds with 20 seconds or a change in color or tone. Apnea of prematurity occurs in premature infants and is often associated with color change and bradycardia. It is common in neonates born at